You are on page 1of 230
eee UNE) A TRAINING MANUAL FOR SOUTH AFRICAN MATHEMATICS OLYMPIADS Published by: South African Mathenatics Foundation Private Beg X173 Pretoria 0001 Tel: +27 (0)12 392 9372 Fax: +27 (0)12 392 9312 Website: www.samf.ac.za © SAMF, Pretoria 2014 ISBN: 978-1-920080-31-0 Author: Prof Poobhalan Pillay Editor: Dr James Ridley Material from this publication cannot be reproduced without the SAMF’s permission Contents INTRODUCTION. i 7 THE PROBLEM: LESSON 1: PRIME NUMBERS AND FACTORISATION 1.1 FACTORS: 46 1.1.1 Definition of a factor. 1.1.2 Multiples. 1.1.3 Multiples and product of strings of consecutive numbers... 11.1.4 The square of an odd number is always one more than a multiple of & 1.1.5 Factor Properties... 1.2 EXAMPLES... 1.3 PRIME NUMBERS. 1.4 NUMBER OF DISTINCT (POSITIVE) FACTORS OF A NUMBER ..... 1,5 DIVISIBILITY RULES 1.6 RATIOS AND AVERAGES . 51 1.7 RATIONAL AND IRRATIONAL NUMBERS 1.8 APPLICATIONS OF LESSON 1...... LESSON 2: HCF AND THE DIVISION ALGORITHM... 2.1 HIGHEST COMMON FACTOR... 2.2 THE DIVISION ALGORITHM 2.3 A PROPERTY OF THE HCF 2.4 COPRIME NUMBERS...nceceenn seve a renee 6 2.4.1 If cis a factor of ab, and cand a are coprime, then cis a factor of b. 2.4.2 Ifa prime pis a factor of a product ab, then p either p is a factor of a or pis a factor of b.......57 2.5 INTEGRAL SOLUTIONS. 2.6 EXAMPLES .ssssnnnne ses 2.7 INTEGRAL SOLUTIONS TO THE EQUATION ax + by = 2.8 APPLICATIONS OF LESSON 2... 60 LESSON 3: CONGRUENCE ARITHMETIC... 61 3.1 THE DIVISION ALGORITHM 3.2 NUMBERS OF THE FORM an+b 3 61 a3 nregens oF THE FoRM where FM)ISA POLYNOMIAL .ssssussnsninonennnn 2 an Sint 3.4 A SPECIAL CASE: NUMBERS OF THE FORM 6q - r, WHERE r= 0, 1, 2, 3,4, 5. coon o 63 3.5 CONGRUENCE MODULO n 3.6 PROOFS OF DIVISIBILITY RULES.. 65 BLT PARITY vss se sents n nnn . 68 3.8 APPLICATIONS OF LESSON 3... LESSON 4: SOME TIPS ON COUNTING ... 4.1 THE PRODUCT RULES IN COUNTING. 4.2 ARRANGEMENTS OR PERMUTATIONS. 4.3 COUNTING ARRANGEMENTS a 4.4 APPLICATIONS OF LESSON 4... LESSON 5: BINOMIAL COEFFICIENTS ‘5.1 NUMBERS OF THE FORM aoe) 5.2 COUNTING SUBSETS.. 5.3 SOLUTIONS TO PROBLEMS 3 AND 4 5.4 BINOMIAL COEFFICIENTS... 5.5 THE NUMBER OF SUBSETS OF A SET. 5.6 PROBABILITY ... 5.7 APPLICATIONS OF LESSON S.... LESSON 6: POLYGONS & PLATONIC SOLIDS. 81 6.1 REGULAR POLYGONS 6.2 REGULAR POLYGONS & CIRCLES 83 6.3 THE AREA OF A REGULAR POLYGON, GIVEN AIDE... 6.4 REGULAR POLYHEDRA & EULER'S FORMULA .. .rssssennmnnininenee nnn BA 6.5 PLATONIC SOLIDS... 6.6 THERE ARE EXACTLY FIVE PLATONIC SOLIDS 6.7 APPLICATIONS OF LESSON 6... LESSON 7: PYTHAGOREAN TRIPLES 7.1 TWO USEFUL IDENTITIES.. 7.2 INTEGERS THAT ARE DIFFERENCES OF TWO SQUARES ....ccicccntnninnnninnininnnenseee BB 7.3 CONSTRUCTING PYTHAGOREAN TRIPLES, GIVEN A SIDE.. 7.4 CONSTRUCTING PYTHAGOREAN TRIPLES, GIVEN THE HYPOTENUSE....-. 7.5 APPLICATIONS OF LESSON 7... LESSON 8: USEFUL FORMULAE... 8.1 SEQUENCES .eeennnnnnninnnnee nnn enn oe 91 8.2 USEFUL SERIES 8.2.1 Sum of the first n natural numbers, 8.2.2 Sum of the first n odd MUMBEFS weenie eon senses 9D 8.3 ARITHMETIC SEQUENCES. 8.3.1 The n term of an arithmetic sequence 8.3.2 The sum of the first n terms of such an arthmetic sequence 8.3.3 Example:... 8.4 SUMS OF SQUARES OF FIRST n NATURAL NUMBERS 24 8.5 SUMS OF CUBES OF FIRST n NATURAL NUMBIRS.. 8.6 GEOMETRIC SEQUENCES... 8.6.1 The sum of the first n terms of the geometric sequence 96 8.6.2 Factorisation of 11" sev ceennnnnn ss aoitstott 7 8.6.3 Factorisation of a” —b" 8.6.4 Factorisation of a” +b" for an odd number a. 8.6.5 Examples: 8.7 OTHER USEFUL FORMULAE: 8.8 The AG (Arithmetic-geometric inequality) 98 8.9 APPLICATIONS OF LESSON 8... LESSON 9: PERFECT NUMBERS 9.1 THE SUM OF FACTORS OF A NUMBER... 9.2 PERFECT NUMBERS. 9.3 A CLASS OF PERFECT NUMBERS. 9.4 CLASSIFICATION OF EVEN PERFECT NUMBERS... 9.5 APPLICATIONS OF LESSON 9 z " sav i 103 LESSON 10: THE FERMAT POINT. 10.1 THE PROBLEM: 10.2 NAPOLEON'S THEOREM... 10.3 APPLICATIONS OF LESSON 10 wns oe ve . . 107 LESSON 11: TRIANGLE FORMULAE 108 11.1 THE SEMIPERIMETER OF A TRIANGLE. 11.2 THE SINE RULE... 11.3 THE COSINE RULE 11.4 AREA FORMULA (1) ssn vette penne oe 110 11.5 AREA OF A TRIANGLE (2) : HERON’S FORMULA. 111.6 RADII OF THE CIRCUMSCRIBED AND INSCRIBED CIRCLE aio ua 11.6.1 The radius of the circumscribed Circle... re snntennnnnnsnne aa 11.6.2 The radius of the inscribed circle. aa 11.7 THE TRIANGLE INEQUALITIES... m1 111.8 APPLICATIONS OF LESSON 11. LESSON 12 (TRIANGLES 2, RATIOS & AREAS). 12.1 WHY IS THE AREA OF A TRIANGLE EQUAL TO"HALF BASE TIMES HEIGHT”? 12 113 12 112 12.2 TRIANGLES BETWEEN SAME PARALLELS AND HAVING SAME BASE HAVE EQUAL AREAS 12.3, A MEDIAN BISECTS THE AREA OF A TRIANGLE 114 12.4. AN ANGLE BISECTOR RATIO. 114 12.5 The Mean Proportion Theorem... a4 114 12.5.1 A useful fact about ratios.. 12.5.2 The Mean Proportion Theorem 11s 12.5.3 Similar Triangles: Two triangles having the same angles are said to be similar. Two triangles are similar if and only if their corresponding sides are in the same proportion. 00.0 116 12.6 CEVIANS 116 12.6.1 Auseful ratio... 116 12.6.2 Ceva's Theorem... a7 12.6.3 An interesting Cevian identity 118 12.7 APPLICATIONS OF LESSON 12... 1g LESSON 13: MANY PATHS, ONE DESTINATION... 120 126 126 LESSON 14: THE INFINITUDE OF PRIMES... 14.1 IDENTIFYING PRIMES 14.2 TO IDENTIFY ALL PRIMES LESS THAN A GIVEN NUMBER. 126 14.3 THERE IS NO LARGEST PRIME NUMBER, 128 130 LESSON 15: USEFUL THEOREMS IN GEOMETRY. 153, PARALLEL LINES ssscacnicnnssovnnoanasiniiniisenson itis . 130 15.2 ANGLES OF A TRIANGLE 130 15.3 ANGLES OF A POLYGON... 1130 15.4 THE ANGLE WHERE BISECTORS MEET. 131 15.5 CONGRUENCY 1B 15.6: ISOSCELES TRIANGLES. 133 15.6.1 If two angles of a ingle are equal, ther the sides opposite these angles are also equal....133 15.6.2 If two sides of a triangle are equal, then the angles opposite these sides are also equal. ....133 15.6.3 Right-angled isosceles triangles... 133 15.7 PERPENDICULAR BISECTORS.....snnnne snes ssn oe 133 15.8 KITES 134 15.8 PARALLELOGRAMS.. 134 134 15.8.1: A quadrilateral is a parallelogram if andonly if its opposite angles are equal 134 15.8.3: A quadrilateral is a parallelogram if andonly if its diagonals bisect each other... 15.8.4: A qua steral is a parallelogram if andonly if two opposite sides are equal and parallel...135, 15.9 THE MIDPOINT THEOREM. 135 15.10 AREA FORMULAE. 135 15.11 THE THEOREM OF PYTHAGORAS 136 15.12 CIRCLES. 137 137 15.12.1 The circumference of a circle. 15.12.2 The circumference of a circle is pi times the diameter. 138 15.12.2 Angle in a semicircle ...... 138 15.12.3 The area of a circle... 139 15.13 TANGENTS TO A CIRCLE... 139 140 140 15.13.1 The radius of a circle is perpendicular to a tangent to the circle at the point of contact. 15.13.2 The two tangents drawn from a point outside the circle have equal lengths... 15.13.3 The kite property of two common tangents... 140 15.14 CIRCLE THEOREMS... 140 140 no 15.14.1 The angle at the centre is twice the angle at the circumferencé 15.14.2 Definition of “ subtends” 141 15.14.3 A diameter subtends a right angle at every point on the circle. 15.14.4 Angles in the same segment are equal. 141 142 142 15.14.5 Definition of a cyclic quadrilateral and concyclic points 15.14.6 The opposite angles of a cyclic quadrilateral are supplementary. 15.14,7: The sides of a cyclic quadrilateral subtend equal angles at the remaining two vertices. .... 142 15.14.8: Ifa line segment subtends equal angles at two points on the same side of it, then the endpoints of the line segment together with these two points are concyclic. 143 15.14.9 If the opposite angles of a quadrilateral are supplementary, then the quadrilateral is cyclic. 143 15.14.10 The tan-chord Theorem: The angle be:ween a tangent and a chord is equal to the angle in the alternate segment. ..sesnmninnnesninins snninnnnnnnnnnnrnnnnnnseennnnnnnenssne: 13 15.14.11: Equal chords subtend equal angles atthe circumference...sunrrvvnnnnnese 144 15.15 APPLICATIONS OF LESSON 15... THE SOLUTIONS 145 INTRODUCTION The mathematics curriculum at South African schoos provides valuable skills in dealing with numbers and studying geometrical objects using graphs, trigonometry, coordinate geometry and so on. However, for various reasons, teaching and assessment are inclined to treat many fundamental ideas superficially, so much so that non-routine problemsare largely downplayed. The major function of mathematics is to make deductions and conclusionsfrom pieces of information that are provided. How best to use the information in order to make conclusions, especially, a desired conclusion, requires a plan, a strategy, an approach. This is what problem solving is all about. Take for example, the following: The average of three numbers is 18. If one of the numbers is replaced by the number 38, then the average of the three numbers is 23. What was the number that was replaced? The solut n to this problem requires no more than what average means. Its a kind of problem that is not done in a classroom. itis a non-routine problem. The learner has to devise a plan to determine the replaced number. This is what real mathematics is al about. It is not easy to display a set of rules that would fit all problems. In fact itis impossible. There are some guidelines however. The SAMF booklets for GET and FET phases provide a number of very useful guidelines. We mention here only one. Replacing a quantity or quantities in a problem, usually an unknown quai but occasionally a known quantity, with a symbol isone of the most powerful techniques available. The information is then translated into equations or inequalities and then solved using the techniques that are taught in traditional schoo! mathematics. The technique can be used in every area of mathematics, and is often used in this manual in the form “Let x be...” Symbols are used mostly to represent numbers, so crdinary operations like addition, subtraction and ‘multiplication can be used on them. In some cases they have special meanings, and in these cases itis vital to know exactly what the symbols mean. For eample, the symbols in a coordinate pair (x;y) refer to certain distances, and one working with them shculd always keep their underlying meanings in mind. ABOUT THIS BOOK South African Mathematics Foundation (SAMF) has created a number of avenues that Olympiad ‘enthusiasts may enter. Solutions to all the competitions run by SAME can be found in its Annual Reports or the SAMF website at www.samf.ac.za. In addition the Foundation has a number of publications, like Enjoying Mathematics and The Love of Mathematics. DVD's have also been made available with accompanying booklets. However, there is quantum jump from SAMO Roune 2 to Round 3, or for that matter, from SAMO Round 2 to the South African Team Competition. Coaches who have been involved in the Siyangoba Training project driven by SAMF which are designed to help bright pupils at eleven centres across the country, have been hard pressed to provide training in this critical area. It is hoped that this book wil fil that need. More than that, it covers all levels up to SAMO Rourd 3, Broadly, the book is divided into three sections, the Problems themselves, Lessons, and Solutions The problems are presented in table form. So are the solutions. Less than half the selected problems are from past Olympiad papers. The problems are not ir any particular sequence, nor is there any direct indication as to which grades they are suitable for. Iisa fact, and this is not particularly obvious, that a significantly large number of Olympiad problems, even up to Senior Second Round, require little formal mathematics to solve. And in a large number of cases, simple primary school mathematics is quite sufficient. Its not uncommon to see Grade 8 learne’s solving Senior Round 2 problems. Each problem is rated according to the cognitive demand required tc solve it. The number in the second column of the table gives a possible rating. The last column provides a reference. A reference lite 12.6 for example, means Lesson 12, section 6, The Lessons have been inserted between the problems znd their solutions. ‘An attempt is made in the “Lessons” to cover the theory that is required for the solutions of most of the problems selected. We make no claim that they arecomplete. But we do believe they will, at least for the present, suffice. Some of the Lessons have little or no bearing to the problems. These are essays that are of independent interest, and which can provide an cccasional diversion. We have tried to present the solutions in such a way that most learners would understand them, but in many cases, we were not able to do so without compromising the quality of the mathematics that ‘accompanied such solutions. At times alternative soutions were presented. Finally, owing to time constraints, we were not ableto include all that we wanted to, but itis hoped that future editions would contain a wider range of probems. Topics like mathematical induction, the pigeonhole principle, the rearrangement inequality, functional equations, advanced topics in geometry ‘and many more, have not been included. We express our deep appreciation to Professor Johann Engelbrecht, Ellie Olivier and other personnel of. the South African Mathematics Foundation for making this work possible and to Professor James Ridley for his insightful and valuable suggestions , which helped to enhance the quality of this manual. Finally, the author dedicates this book to his beloved parents, Sivananda and Saraswathi. Poobhalan Pillay (Academic Coordinator: Siyangoba Regional Training Programme, Emeritus Professor in Mathematics, University of KwaZulu Natal) THE PROBLEMS 300 [WE QUESTION REF 4] | Factorise 3-1 into prime factors. 7 | ® | Which of the following numbers is the smallest? 0,2013 ; 0,201 ; 0,0201; 0,02013; 0,02. | | Themba and James sell cakes at school socials. At the first social, they sold 50 cakes, and at the second social they sold 58 cakes. What was the percentage increase in their sales? ‘A group of children see a herd of cattle inthe veld. They count the total number of legs and the total number of ears of the cattle The difference between these two numbers is 92. How many animals are there in the herd? The desks in a classroom are lined up in straight rows. Vusani's desk is in the third row from the front and the fourth row from tke back of the classroom. His desk is also the fourth from the left and the sixth from the right. How many desks are there in the classroom? a Pieter and Jacob share a packet of sweetsin the ratio 7 :5. Pieter gets 14 sweets more than Jacob. How many sweets were there in the packet? In the figure alongside (not drawn to scale),all angles are right angles. What is the perimeter of ABCDEF? a A fraction that lies between = & 19, 32, 38, 38.37 56’ 56 56°56’ 56 a Calculate F. Mary has three types of toys: teddy bears cars and jets All her toys except 21 are jets. All her toys except 23 are teddy bears. All her toys except 26 are cars. How many jets does she have? w 12870456 is an 8-digit number, where a and b are not zero. The number is divisible by 18, What is the maximum possible difference between a and b? w ‘A bead worker is threading beads onto a straight wire; he has four green beads and two red beads and will use them all. How many different arrangements can he make? 15 | @ | Shoba and Shanti have to dust all the ornaments (100 of them!) in their house every week. If Shoba does it on her own, it takes her 1 hour and 40 minutes. if Shanti does it on her own, it takes her 3 hours and 20 minutes. If they do it together, how long will it take them? |G | ifdanuary 1° 1985 was a Tuesday, how many Tuesdays were there in 1985? 1 [| How many zeros are there in 300%", when written out in full? IE |_| The three-digit number 7a2 is divisible byi1. Determine d. @ |S | Calculate 1 (24+4+6+--+50)-(143+5+- +49) 822 B/S nx2 Asm gets larger and larger, what does thevalue of "gets closer and closer to? nt A bathroom floor is cove'ed with square tiles. The floor is 5 tiles wide and 8 tiles long. If one ofthe floor tiles is chosen at random, what is the probability that itis at the edge of the floor? 2 {eS 2 circle is divided into four sectors. ZA=5 4C, 2D=228. Angles B and Care supplementary. Calculate the size of ‘The strings meet at a point P. What is the height of P above the ground? 7 | | ifyou write the integers 2, 3, 4, 6, Bn every possible order to form S-digit numbers, how many of these numbers will be divisible br 11? | | Find the value of k+ Vif k and / are positive integers and k +/+ kI= 54. zie (@) What is the smallest positive integer that leaves a remainder of 3 when divided by 10 and a remainder of 4 when divided by 13? (b) What is the smallest positive integer that leaves a remainder of 11 when divided by 25 and a remainder of 12 whes divided by 36? @ | ™ | How many positive integers n are there such that n + 3 is a factor of n? +7? @ | | Prove that every Pythagorean triple contains at least one multiple of each of 3,4 and5. | 3.5 [17 | The largest odd factors of the integers from 1 to 2" are added. Calculate the sum in terms ofr, le Points P, QR, and Sare marked on the sides of square ABCD so that each side is divided in the ratio 2 : 1, and therefore PQRS is a square. Calculate the ratio of the area of PORS to the area of ABCD. The natural numbers are written in sevencolumns 1 2 3 4 5 6 7 8 9 10 11 12 13 14 is 6 7 A square is drawn around a 2x2 block of rumbers in the grid. If the sum of the four numbers is 312, what is the number in the top left square? ‘A block of eight flats with a stairway in the middle is situated on the top floor of a building. The positions of the flats are shawn on the sketch below. There are windows on all sides of the building which afford good views. Twice as many people have a southward view as there are people who :an look eastward. Those with a westward view number only one-third of thase who can look south, while the few who have a northward view number only half of those who can look east. Altogether 20 people live in the eight flats. How many occupy each flat, given that no flat is vacant? Explain your ‘answer and show the number of occupan's on a sketch. South AB is thediameter of the semicircle and AB = 10. F the area of AABC is 11, find the perimeter of, ‘ABC. = [a For which one of the following statements does there exist an x which makes it true? (a) 2x = 3 (mod 12) (b) 3x=7 (mod 12) {c) 6x = 11(mod 12) (d) 5x29 (mod 12) (mod 12). 3S uv a Fareeda would ike to become an Olympic sprinter. Her younger sister Sumayya would rather play football, ut helps Fareeda byracing against her. When they tried the 100 metre dash, Fareeda crossed the winningline when Sumayya was stil 20 metres short of it. Fareeda wanted something more challenging, soit was agreed that Fareeda would start 20 metres behind the starting line. Trey both ran exactly the same speeds as in the first race. Where were Fareeda and Summaya when the winning line was crossed by whoever arrived at it first? Explain your answer. ‘Aster, Baster, and Caster are three villages, as shown in the diagram below, where the straight lines represent the only roads joining the villages. The figures give the distances, in kilometres between villages. 8 f 2 ¢ Anew fire station is to be built to serve al three villages. It is to be on a roadside at such ‘a position that the greatest distance that she fire-engine has to travel along the roads in ‘an emergency at one of the villages, is as small as possible. Where should the fire station be positioned? Locate your point on the diagram, and explain why no other position is satisfactory, a A child’s age, increased by 3, gives a perfect square, and when decreased by 3 gives the (positive) square root of that perfect square. How old is the child? Calculate the value of: 242427 42424" a If 10 —1is written out in full, find the sum of the digits. 7 J" the diagram, the congruent circles are tangent to the large square and each other as shown; and their centres are the vertices of the small square. The area of the small ‘square is 4, Find the area of the large square. he MAA a I wrote down the integers 25, 26, 27,.., 208. How many digits did | write down? a Ifthe number A12345678 is divisible by 45, determine the value of A+ B. Py Find the value of 2013" ~2(2000}(2013) + 2000" is a Calculate the value of 2013-2009+2005-2001+1997-1993+--#29-25. | In the diagram, the sides of the largest triangle are divided into three equal parts to produce smaller congruent triangles as shown. This process is repeated for the smaller triangle on the bottom right. If the area o' the largest triangle is 81, what is the total area of the shaded triangles? a ‘A bag contains 65 marbles of the same size. There are 20 red ones, 20 green ones, 20, blue ones, and another 5 that are either yellow or white. Lindiwe removes marbles from the bag without looking. What is the smalest number of marbles that she must remove to ensure that she has 10 of the same colour? & Determine the number of pairs (x; y) of integer solutions for 2" -3"” = 55. a In how many ways can five students A, B,C, D and E line up in one row if students B, C and D must stay together? Two tangents are drawn to a circle from apoint A, which lies outside the circle; they touch the circle at points B and C respectively. A third tangent intersects AB in P and AC in R, and touches the circle at Q. If AB = 20 and PQ = 3, find the perimeter of A APR, € 15.13 a The finction fix) ie defined for all real numbers x Wf flab) = Fla)f(h) foralla and h, and f{1) = 2, find the value of #12013). w Jack, John and James are identical triplets. It is impossible to distinguish them by appearance. Jack and John always tell thetruth, but James always lies — everything he says is false. You know that the triplets ars between 20 and 30 years old, 20 and 30 included. One day you meet two of the triplets and ask them how old they are. A One says “We are between 20 and 29 years old, 20 and 29 included”. The other makes the following statements: “We are between 21 and 30 years old, 21 and 30 included” and “One of us present is lying". How old are they? 9 & John takes 300 steps to walk from point Ato point B ina flat field. Each step is of length 1 = meters, and he makes a 90° turn after every step except the last one. He makes 99 E ry step except left turns and 200 right turns in total. He stops at point 8. What is the maximum possible distance from A to B? 71 | Determine the volume of a octahedron wose edges have length 3y2. (An octohedron is a 3-dimensional objectbounded by 8 congruent equilateral triangular faces. As illustrated it can also be viewed as two pyramids with square bases that are stuck together.) 1G |S | What is the largest possible area of a quadrilateral with sides of length 1, 4, 7 and 8? Isai TGS |S | Erica noted that a train to Muizenberg took & minutes to pass her. A train in the opposite direction to Cape Town took 12 minutes to pass her. The trains took 9 minutes to pass ‘each other. Assuming each train maintained a constant speed, and given that the train to Cape Town was 150 m long, what was the length of the train to Muizenberg? 17 | 8 | in triangle ABC the angle at A is 60° and the inscribed circle of ABC touches AB at the point D. If AD = 5 and DB = 3, find the length of BC. Pegs are nailed into a board 1 cm apart as shown in the diagram. An elastic band is stretched over five pegs as shown, What isthe area of the pentagon so formed? 5 ‘Two opzosite corners of an 8x8 grid are removed, 50 62 squares are left. You have thirty-one 1 x 2 dominos. Place the dominos on the grid in such a way that all 62 squareson the grid are occupied. =" a ‘Suppose I try to count to a billion, that is : 000 000 000. If count one every second, without stopping to rest, how long wouldit take? Give your answer to the nearest year. w Imagine a long string wound tightly around the Earth at the Equator. The string is then lengthened by 10 metres, the ends knotted together, and the longer string is raised above the ground to forma circle which is at the same height above the ground throughout the circumference. How high s the string above the ground? LONGER STRING —> The product of the ages of a group of children whose ages are all between 12 and 20 is 10 584 000, How many children are therein the group? i Two points A and C lie on the same side of a straight line. Find a point X on the line such that the sum AX + XCis as small as possible. "i c x The square ABCD has sides of length 6 units. Mis the midpoint of AB and P is a variable point on BC. Find the smallest value of DP+ PM. la & ‘ABCD isa rectangle, and P is an arbitrary point in its interior. Determine PA in terms of PB, PC and PD. Between 12:00 and 13:00, there are two times when the hands of a clock are exactly at right angles. How many minutes apart are these two times? ‘ABEG is a square. Cis an interior point of the square such that ZCEG= ZCGE Prove that ABC is equilateral. a BE a Each of the 64 squares of a chessboard cantains either 0 or 1. Prove that amongst the 18 row/column/diagonal sums, at least three are equal. E i Let a, band ce the sides ofa triangle, ard p, q andr be numbers satisfying p+q+r=0.Prove that a’ pg +b'gr-+c’m <0. The sides of square ABCD all have length = unit. M and K are points on BC and CD respectively, such that the perimeter of triangle CMK is 2 units. Find Z MAK. 4 2 a ° K w You are asked to pick the first, second and third athletes in a race in which there are 12 athletes. What is the least number of choces you have to make to ensure a win? If you are given twenty guesses, what is the probability that one of them is correct? a Eight books are to be arranged on a shelf. In how ways can they be arranged? BE Fight hooks are ta be arranged an a shelf sa that two particular books are to be at the ends. In how many ways can this be done? How many 5-digit numbers greater than €0 000 are odd? You wish to create a string of 10 numbersusing only 0 and 4. How many such numbers can you create? 3 Derive a formula for the sum of all properfactors of p”, where pis a prime number. (A proper factor of a number is one that is diferent from the number itself). 127 [3] Explain why any (natural) number can be sxpressed as the product of a power of 2 and ‘an odd number. 2s | | Factorise 2014 — 1 as a product of prime 1umbers. 13 22) 3 | in the figure, ABDC and EFGD are squares If CE= CD and ZDEC=70", find ZBDG 15.6 380 | UA right triangular prism has edges in the ratio 3:4:5:20. If the volume is 202,5 units, find the length of the longest side. L Tai |W Prove that the square root of any prime number is an irrational number. 132 | 1 | Pick any natural number n, no matter how large. Then we can find n consecutive numbers, none of which is a prime number. Prove this. Discuss the implications of this, fact with regard to the infinitude of primes 153 | 3 | Use the Division Algorithm to find the HGF of 1407 and 4757. 22 13! || Determine an integral solution ~ that is, asolution where x nd y are both integers of | 2.5 the equation 1407x + 4757y = 67. B| Find an integral solution of 125x + 36y = 1. HE |S | Prove that the product of the HCF and LCU of two numbers is always equal to the product of the two numbers. we a n)_nln—1)___(")_nln—1\in—2) Verify that and |, |-> “= ev) 9 (3) wpe a How many 3-element subsets does a set faving 8 elements have? a How many 98-element subsets does a sethaving 100 elements have? ey ly bl sty to explain why ( 2?) =(709 8 jgument, try to explain why | 4 =|" TEE] | Aset has 300 elements. How many subsess does it have? 54 2 | |i. Prove that the sum of two odd numbers is even and that the product of two odd | 11 numbers is odd. 2. Show that if the sum of two integers is odd then their product is even, 12 | @ | There are 100 soldiers in a detachment, ard every evening three of them are on duty, Can it happen that after a certain period of time some soldier has shared duty with every other soldier exactly once? 184 | | Ona chessboard, a knight starts from square al, and returns there after making several moves. Show that the knight makes an evan number of moves. (Square a1 is the bottom left corner square in a chessboard.) ES | | A certain country has coins having values, 1, 3, 5 or 25 cents. Can one make change ofa_| 3.7 25 cent coin, using a total of ten coins each having a value of 1, 3, or S cents? 1H | @ | Given two integers a and b, consider the number abla—b). Can you determine ifitis, even or odd? a |B | Katya and her friends stand in a circle. It tarns out that both neighbours of each child are of the same gender. If there are five boysin the circle, how many girls are there? 1S |G | Seven gears are placed on a plane, arranged in a circle, such that each gear engages the ‘two gears next to it. Can all the gears rotate simultaneously? HB | © | List the first seven triangular numbers 150 | 3 | Consider the following sequence of “triangles” built up from dots ras et rat 1. List the number of dots in the first five figures. 2. How many dots are there in the 100” fare? 3. How many dots are there in Fig. n? n(n -1) 4. Why are numbers ofthe type “called triangular numbers? 5. Using ideas from above, can you suggest why numbers of the form 1’ are called square numbers? we How many factors does 180 have? w Find all numbers less than 50 that have eactly four factors. TE | | For each postive integer n let a(a) dence the sum of all the Factors of n, Show that ‘o(r)=n-+1 ifand only if mis a prime number 1) | caleulate o(3600). a [| let S= =a SP = ae w tom ters (alternating in ig). BS Find al values ofm for which Sis a perfect square. ra r 2018 setermine n. 014 7S | Let ABCD be a square and a point such that A and X are on opposite sides of CD, The | 12.3 lines AX and BX intersect CD in Y and Z respectively. Ifthe area of ABCD is 1 and the area of XYZis¥, determine the length of YZ. TEE] Tat pea prime mammber, Find all postivelntegers Xand y such that + pray a 2a | How many positive factors of 60 are also multiples of @? mye ; E The measure of angle ABC is 0", AD bise:ts angle BAC and DC bisects angle ACB. 152 Calculate ADC EI || Ayanda, Gert, Tina and Precious are goodiriends, Precious had no money, but the others did. Gert gave Precious one-fifth ois money, Ayanda gave Precious one-quarter of his money, and Tina gave Precious one third of his money. ach gave Precious the same amount of money. What fraction ofthe group’s money does Precious now have? 152 | | How many ofthe numbers 1, 2,3, 4,... 2014 are divisible by neither 11 nor 137 TS]? | calculate orbserdse i (xa) ar tbe For verre E/E ABCD isa square and ADE isan equilaterd triangle, with E inside the square, Calculate | 15.6 ABE HEED | A square is changed into a rectangle by increasing two ofits sides by p%, and reducing two others by p%. The area is reduced by1%. Calculate p. mye Five animals A, B,C, D and € are either welves or dogs. Dogs always tell the truth, whereas wolves always lie. A claims that 6 is a dog. C claims that O is a wolf. € claims that ‘Aisa dog. 8 claims that Cis a wolf. D clains that 8 and E are of different kinds. How many wolves are there? a Isosceles right-angled triangles are constricted on the sides ofa right angled triangle, as shown below. A capital letter represents the area of the triangle. Prove that A=B+C. ‘188 | 3) When Fanie bought his train ticket, he received R3.20 in change. He noticed that for each coin in his change there was exactly one cther coin of the same value. What was the coin of smallest value in Fanie’s change? 1878 Vp Qe 2GEAP=1, tind m in terms of n TBE} 3 Goolam has 2 sisters and & brothers. His sster Fawala has sisters and B brothers. Determine the product of S and B. In music a demisemiquaver is half of half of half a crotchet, and there are four crotchets in a semibreve. How many demisemiquavers are there in a semibreve? ‘The sum of two numbers is 2, The differeice between them is 4, What is their product? Prove that the real solution a of x’ +px+q=Osatisfies 4aq 8 c M E 1 | | Each of the angles ofa triangle, when expressed in degrees, is a perfect square. Find the three angles. 1S |G | Calla four-digit number balanced ifthe sum of the two left-most digits equals the sum of the two right-most digits. For example, 2754 is balanced, How many balanced numbers are there between 1000 and 2000? 18 | 4 | There are exactly four positive integers n such that (#7) isa factor of (n¥1)*. Find the largest such n. 187 | @ | Ten (not necessarily distinct) integers have the property that if all but one of them are added, the possible results are: 82, 83, 84 85, 87, 89, 90, 91, 92. What is the smallest of the integers? ms ‘Azircle is inscribed in a triangle whose sides are 46, 40 and 48 cm respectively. A smaller circle is tangent to the two equal sides of the triangle and tothe first circle. Find the radius, in em, of the smaller circle 18 | | Find the sum of all numbers x that satisfy the following system of equations: yar? 5x45 z 382 || How many zeros are at the end of the product Ne= 1x 23% x 199 x 200? TET | Win the figure, ABCD and EFGH are rectangks. Calculate the area of EGFH. 7510 A E 8 LOT rs 8 © #2) 4 ifm and n are counting numbers such that m>n and m+n ='8-+/60 , determine m-n. WO [txty=a, yrz=b, z+x=C, deternine xin terms of o band. w Ba In the diagram on the left, there are 14 squares of all sizes. How many squares are there all together in the diagram on the right? 185 | 3 | Agnes says “Billy always speaks the truth”. ily says, “Agnes is lying”. Charlotte says. “Agnes and Billy are both Wing”. Which one of the three is telling the truth? = feasts ts Fy. determine 142424242... interms of x. 3°25 4981 157 | | Jack gave ill a 10 metre lead in a 100 metre race and Jack was beaten by four metres. What lead should Jack give Jill n order that both finish the race together, if their speeds stay the same in both races? TEE | [A cube of side length 1 is cut across a diagonal to form two identical wedges, one of which is shown in the diagram. What is the total surface area of the wedge? 3/4 | fx =3, determine the value of 86 Ox 10008 4 2X) 289] © | Three circles, each of radius r, are inscribed in an equilateral triangle with sides of length | 15.13 1. If each circle touches the other two, aswell as two of the sides of the triangle, calculate r. 201 | 6 | P(x) is a polynomial of degree 5. if r)=2, pe)=3, Prai=4 & P46)=2, determine PC) WD |B | The average of three numbers is 18. fone of the numbers is replaced by the number 38, | 1.6 then the average of the three numbers is 23. What was the number that was replaced? ‘Two ants start at point A and walk at the same. pace. One ant walks around a 3 cm square whilst co tHe other walks around a 6 cm by 3 cm rectangle. What is the minimum distance, in centimetres, sem that each ant must cover before they meet ten again? me The letters A to | represent numbers. The pee numbers are added vertically and horizontally to give the numbers in the last row (17; P:Q) Dye Te faa and the last column (17; 14; 15) Calculate P + Q, G PHI fas: a7 |p fa Hi mle Determine the angle x in the regular octagon alongside. 30 | Mary was given the task of removing all multiples of 2 and 3 from the set of numbers from 1 to 100. How many numbers were Eft? (2 [ABCD is a rectangle. E and F are midpoint: of AB and AD in that order. Ifthe area of triangle BEF is 10cm?, calculate the area of rectangle ABCD, in square centimetres. D c a The areas of the two rectangles in the figure are 80 cm? and 60 cm’, a The letters A, B and C denote the areas of the regions containing, them. Calculate A-B. 3 [1000 dots are evenly spaced on the circunference ofa circle. They are numbered successively from 1 to 1.000 with dot 1 0C0 opposite dot S00. Which dot is opposite dot 657? "ifthe length to breadth ratio remains the same, whether a book is opened or closed, find the ratio of length to breadth, | in the following calculation, a and b represent digits. 79287 2h if =2b7,, determine a+b. Three rulers and one pencil cost the same as two erasers. One ruler, two pencils and three erasers cost R25. I the price of each item is a whole number of rands, what is the price of an eraser? | Each male honey-bee has a single female 2arent whilst each female honeybee has both a male and a female parent. In the 10" generation back, only, how many ancestors does a male honey-bee have? 8 | Here is something you can try out on a friend: You write down something on a piece of paper, fol folded paper into his pocket without lookng and ask your friend to put the ‘Ask your friend to write any three-digit number he likes on a piece of paper, However, 31 the first and last digits must not be the sane. ‘Ask him to write the number backwards, and subtract the smaller one from the larger. The answer must be another three-digit number. If not, he must put a zero in the front. (For example, for 46 he must write 046). Then ask him to add this number to the three-digit number obtained by reversing it. Let him compare his final answer with the number written on the paper in his pocket. They will be the same! Explain why. w Let P be a point between two parallel lines L; and Lz. Ais a point on L; andX and ¥ are points on L2, as shown, Prove that the aiea of triangle AXP is smaller than the area of, triangle AYP, & A x v ae & How many numbers in the sequence 1, 2,3,..,99999 have an odd number of odd digits? w ‘Our local baker sells small buns for R3 each, 7 for R10, or R18 a dozen. My mother gives me R100 to bring home 60 buns and tells ne to keep the change. | want to be able to bring home at least 60 buns and have as much money left over as possible. What is the most change I can keep? ‘Ashana and Bridget each have a/jug of 1 tre of water. The first day, Ashana pours 1 me of water into Bridget’s jug. The second day, Bridget pours 3 m€ into Ashana’s jug. The third day, Ashana pours 5 mé into Bridge’s jug, and so on, one of them pouring 2 me of water more than she got from the other the previous day. What is the volume of water that Ashana will have on the 101* day? Ge your answer in millilitres. a ‘What is the largest number of diagonals that can be drawn on the faces of a cube so that no two of the diagonals have a common foint (including end points)? How many different isosceles triangles having perimeter 25 cm are there, ifall the sides are a whole number of units? ‘A group of students organized a car washto raise funds. Some customers had a basic car ‘wash for R25 each, while the rest had a vecuum-shine for R35 each. A total of R880 was raised. What Is the minimum number of customers that would make this possible? a Ina group of 40 students, 20 play tennis, 19 play volleyball and 6 play both tennis and volleyball. Calculate the number of students who play neither tennis nor volleyball. 32 23] 7 [A point P has distances 3, 4 and 5 from the vertices of an equilateral triangle, as shown. | 15.11, Calculate the length of one side of the equilateral triangle. ea 22/15 | Let Nbe the common sum of a 3x3 magicsquare. (If you add the numbers in any row or column or diagonal, you will get the sameanswer). Prove that the middle number is NB. E | Find all pairs of integers (x, y) that satisfy che equation 22 +397 = 2012. ‘The figure shows the plan of a town where all the street blocks are square. In the middle of the ‘town is a park with a diagonal road through it. Gloria walks every day from her house H to her school , always taking one of the shortest routes. Determine the number of different routes she S cantake, 227 | @ | An isosceles triangle has sides of length 2,2 and x. For which value of x is the area of the | 11.7 triangle a maximum? 2 | (| Find all natural numbers n such that n(n+2013) is @ perfect square. i #8 \6 | Show that the quadratic equation x* +7x—14(q" +1)=0, where q is an integer, has no integer root. 230 | | Without using a calculator, determine all positive integers x and y such that xy =1999(x-+y). (Note that 1999 is a piime number.) ale ABCD is a rectangle such that AD = 2A, FAC =a\/S , calculate the perimeter of ABCD in | 15.21 terms of a, "| Six numbers are represented by a, b,c, d,e and f. The average of a, b, cand dis 10. The average of b, c, d, e and fis 14. Iffis twice the value of a, calculate the average of a and e 33 w Water flows through a network of pipes from A to B. Five taps are on the network as shown. Each tap can be opened or closed to let water through or stop its flow. There are 2° = 32 different ways of setting the taps. How many of these 32 ways will always allow the flow from A to B? w From the numbers 1, 2, 3,...500, a sequerce is formed by deleting numbers so thatno | 3.5 ‘two of the remaining numbers have a sum which Is a multiple of 7. What is the maximum number of numbers in this sequence? w/e a, 4 Steck, where a, 0,, dy. a, are natural numbers, determine the aa, smallest value of n. BE] 1S | Let Qbe the Fermat point of triangle ABC (We assume that allthe angles are less than | 10 120 degrees). That is, of all the points inside the triangle, the sum 3 QA+QB+QC < PA+PB+PC for al other points P inside the triangle. Prove that 4 1 QA +08 +0 plese +¢7)+2V3A where o, b and care the lengths of the sices BC, CA and AB respectively, and Ais the area of triangle ABC. 22/4 Given the equations 7 +y*=2, yoxte determine the value or values of c so that: (a) There are exactly two pairs of nurbers (x; y) that satisfy both the equations. (b) No pair of numbers satisfies both equations simultaneously. (c) There is exactly one pair of numbers (x; y) that satisfy both the equations. EU Find the shortest distance from the origin to the straight line having equation x+y: 232 | 3 | Find the units digit of 1h+-21+31+ay---+20131 20 | | How many squares of any size can be forned by joining the dots in the following regular arid? 34 a Two cars race around a circular track at constant speeds, starting at the same point. If they travel in opposite directions, they meet every 30 seconds. if they travel in the same direction, then they meet every two minutes. Ifthe track is 1800m long, what is the speed of each car? a Your brother will be x years old in the year 2°. In what year was your brother born? ‘A sequence has first term 12, after which every term is the sum of the squares of the digits of the preceding term. Thus the second term is_1°+2° = Sand so on. Find the 2015" term of the sequence a ‘Suppose a dartboard has only two possible scores, namely, 4 and 9. That is, a dart lands ata point where the score is either 4 or 9,So scores like 4, 8, 9, 12, 13 and so on can be obtained, while scores like 1, 2, 3, 5,6, 7 znd 10 cannot be obtained. Show that there is @ number beyond which all numbers can be obtained and find the largest number that cannot be obtained. BS | | Note that 39 = 3194349, (a) Find all two-digit numbers that arz equal to the product of their digits plus the sum of their digits. (b) Find all three-digit numbers havirg the same property, 2 | @ | The perimeter of a rectangle is 24 cm. Find the maximum area of such a rectangle. 247 | | A bucket is 32 cm in diameter at the top and 20 cm in diameter at the bottom. Find the capacity of the bucket in litres, if itis 21 om deep. Two candles of the same height are lit at the same time. The first candle is completely burnt up in 3 hours while the second cancle is completely burnt up in 4 hours. After how many hours is the height of the second candle equal to twice the height of the first candle? 35 | The numbers 1 to 9 are placed in a 3x3 grd. The numbers in each adjacent pair of ‘squares are subtracted, the smaller from the larger, and 9 s 1 _ | the sum ofthese twelve numbers is calculated. In the ‘example alangside we have Row1: 4.4 Column4: 2,5 7 4 8 | Row2: 3,4 Column2: 1,1 Row3: 1,3 Column3: 7, 2 Total = 8474 +742+9=37. 2 3 © | show howto arrange the nine numbers so that the sum is as large aspossible. Prove that no larger sum exists. 250, | A structure is built with identical cubes. The top view, the front view and the side view are shown. What is the least number of cubes required to build this structure? The top, front, and side views are: FRONT VIEW SIDE VIEW TOP VIEW Find three different positive integers, thesum of any two of which is a perfect square. 2 ‘A camel sits next to a pile of 3600 bananas at the edge of a desert. He has to get as many bananas as possible across the desert, wich is 1000km wide. He can carry only 1200 bananas at any one time. To survive he hes to eat one banana for each kilometre he travels. What is the maximum number of sananas that he can get to the other side of the desert? & Prove that 2° +-5*™ is not a prime numer. In SABC point Dis on AC and AD = AB. tis also known that ZABC— ZACB=30" Calculate ZCBD. 36 a ‘The perimeter of the region A is 25cm. Calculate the area of the circle. ‘An integer K, when divided by 6, 5, 4, 3, and 2, leaves remainders of 5, 4, 3, 2, and 1 respectively. Find the smallest such K. Before his last Maths test, Sunil’s average was 33%. In his last test, he scored 40%, which increased his average to 34%. What musthe score in the next test to increase his average to 35%? a Fred puts 11 plastic bags inside another pastic bag. Each of the 11 bagsis either empty or it contains another 11 bags. Altogether 6 bags contain other bags. Of all the bags, how many remain empty? Mr Mahlanyana had 9 children and 31 grandchildren. In his will, he left a sum of money to each grandchild. Each girl was to get R7 more than each boy. All 31 grandchildren were alive when Mr Mahlanyana died anc the total sum due to them was R470. Of this sum R74 went to Mrs Zweli’s children. (She was the eldest daughter.) How many daughters did Mrs Zweli have? a In the figure, DA= DC and DB = DE. 2BOC =40'.Find ZABE. a 2 5 w ‘One of the angles of a rhombus having sice 2 is 30 degrees. Determine its area, e e Is lo 37 15.6 155 (a) In the above figure, prove that BA (b) Given two lines @ and b which are not parallel, and a point B between them, show how to determine, using only an unmarked ruler and compass, points Aon a and Eon b so that ZABE =90" and BA = BE. a 1 | Let any 2n points be chosen on a plane sothat no three of them are collinear, and let any | 11.7 of them be coloured red and the other m blue. A line is drawn from each point to exactly one point of the other colour. Prove that itis always possible to do this in such a way that no two of the resulting n lines intersect. 251 [7 T A set S of 2n + 3 points is given on a plane, so that no three of them lie ona straight line, | 15.14 ‘and no four of them lie on the same circle, Prove that itis always possible to find a circle | .4 C that goes through exactly three points cf S, and that splits the remaining 2n parts in half, that is, n are inside the circle and n are on the outside. 265 || APBis an arc of a circle (see diagram belon). Prove that AP + PB is amaximum ifand 15.14 only if P=C, the midpoint of the arc. 123 S. Lz 26 | | ifevery point in the plane is coloured, using one of two colours, prove that thereisan | 15. equilateral triangle all of whose vertices have the same colour. 15.6 112 15.14 261 |5||TABC has CA=2 and CB=1. What is the maximum value of ZA? 38 wo Ifall the diagonals of a convex polygon with m sides are joined, and no three diagonals are concurrent, prove that the interior of the polygon is then partitioned into in) (n-1 a)"{ 2. | regions. (We may assume that 24.) (ire?) ithe numero wayrobad may be chosen om beds See Leson ‘A ABC is acute angled. The semi-circle having centre on BC, and touching AB and AC has radius r,. Likewise r, and r_ are defined. Let r be the radius of the inscribed circle of, AABC. 2, e aaa Prove that 7+ =-+— In triangle ABC, AB = AC, and D is the midpoint of AB. Let be the centroid of A ACD and O the circumcentre of ABC. Prove that GOL CD. (The centraid of a triangle is the point where the three ‘medians meet, and the circumcentre is the point where the three perpendicular bisectors meet. Its also the centre of the circle that passes through A, Band C.) a Find all integers m and n satisfying the equation m’ —n’ =2m’ ~3m+1. m/e 1p Ca PMe= a) «(determine the vatueot (2—CO—a) (o-q(d-2) (a—b\(c-a) The sum of certain integers is found to be 2643. It was discovered later that the digits of ‘one of the three digit numbers was reversed (abc instead of cba). Which of the following isa possible correct sum?: 2821, 2733, 2742, 2614, 2568. Determine all numbers whose square is equal to the cube of the sum of their digits. 39 5 ‘A point A on aalane is connected to 16 other points, and every two of tre 16 points are also joined. The figure on the left shows some of the connected points. Three colours, namely, red, be and green, are used to colour each of these segments. Show that amongst all the triangles so formed, there is at leas: one whose sides all have the same colour. 5 ‘Two convex pentagons lie in parallel planes and each vertex of ‘one is joined to each vertex of the other. These 25 line ‘ segments together with the ten sides of the two pentagons are coloured either ‘ed or blue. None of the triangles so formed have all three sides having the AA same colour. Prove that all ten sides of the pentagons must then be of the same colour. | 15.14 23 ‘ Let AB...C be a regular n-sided polygon having sides of | | 11.4 length 1. Consider triangle ABC and the n— 1 line segments that radiate from A. Let two of these, joining adjacent vertices, intersect BC atXand¥. Prove that AX.AY = XY m/s ? a, i P Prove that if — —— +, where — is in its simplest form, then p q 234 1334 1335 q is a multiple of 2003. (Note that 2003 is aprime number.) 28| 16 | show that m* +(m+1)* =n? +(n+1)’ has no solution for positive integers m and n. 280 | 3 | Consider the number 123456789101112131415...585960. Delete 100 digits from this number, without altering the order of the digits, so that the remaining number is as large as possible. mf Find all integers x, y and z such that x° +)°+2° 40 Let a be any non-zero real number. Determine all solutions of the simultaneous equations i Determine all geometric sequences x; y; 1 of positive integers whose sum is 111. ABC is a triangle with ZC=90" and AB =4. A square CDEF, with D, E and Fon CA, AB and BC respectively, is drawn. If CF = 1, calculate AD, given that AD > 1. & Let ABCD 2e a convex quadrilateral. Prove that ‘AB? +CD' = AD? + BC? AC LBD. Find all pairs of positive integers m and n such that the sum of all the integers from m to ‘m+n (inclusive) is 120. ‘50 points are chosen inside a convex polygon having 80 sides, such that no three of the fifty points are collinear. The polygon is cut into triangles such that no two triangles intersect, and the vertices of all the triangles are just the 50 points and the 80 vertices of the polygon. How many triangles are thers? Find all positive values of n for which 3*..3" +3"is a perfect square. Ten different numbers are chosen from the set {1, 2, 3, 4,5, these ten numbers we can find @ and b such that s i s 2. 90). Prove that amongst un mo) Given AABC and a point O inside the triangle. | 15.6 Ait ZBAC =45,, ZABC =30', and ZOAB= ZOBA=15° calculate ZAOC. wife Inthe figare alongside, calculate 0 me i Arational number — has the form xxxx...x, XXX...167Xxxx..., where each x stands for a digit from 0 to 9. Determine the smallest value of n. 2 | 16 | Consider the sequence whose n term is f, =n’ +20. So the sequence is: 21, 24, 28, 36, 45, 56, 69, 84, 101, 120, 141, 164, 189, 216, Now construct the sequence made up of the HCF of consecutive terms. So the n™ term d, of this new sequence is: 3,1,1,9,1,1,3,1,1,3,1,1,27,. Find all possible values of d, . 284 | 4 | Find all natural numbers whose squares end in 444. Ble a In AABC, AB = AC and BAC =100 . The tisector of angle B meets AC at D. Prove that AD + DB = BC. 2 [az 5 w Reduce the fraction 2244851485148514627 8118811881188118000 to its simplest form. ‘A sculpture consists of three large cubes stacked one on top of another without overhanging. The largest cube has edge 3metres and its base is on the ground, The next, cube has edge 2 metres and the top cubehas edge 1 metre, The exposed surface is to be painted blue Each tin of paint will cover ten square metres. How many tins of paint will be needed? & Louise's house has a staircase with twelve steps. She can go down either one step or two steps ata time. For example, she could ge down 4,2,12,4,1,1,2,4,2,4,1. In how many different ways can she go dewn the staircase? a ‘A supermarket stocks 4 kinds of bread, 5 tinds of meat and 3 kinds of cheese. It sells, three types of sandwiches, each made from one kind of bread with either one kind of meat or one kind of cheese or a slice of meat and a slice of cheese as filling. How many different sandwiches are on sale? a Ina recent election, six candidates stood and a total of 51 880 votes were cast. The winning candidate beat the others by 1336, 7085, 15 333, 15 654 and 17 102 votes. respectively. Candidates lose their deposi: if they fail to get more than 5% of the total number of votes cast, How many candida:es lost their deposits? w Two cars race around a circular track, in opposite directions, at constant rates. They start at the same point and meet every 30 secends, If they move in the same direction, they meet every 120 seconds. If the track is 1820 metres long, what is the speed of each car? w ‘Shopping at a supermarket, I spent R6,000n brown rice and R9,60 on white rice. | brought home 20 kg of rice altogether. The prices per kilogram were exact numbers of cents, and white rice costs 5 cents more tran brown rice. How much of each type of rice did | buy? Three circles with centres A, B and Care tangent to each other as shown below. Each circle has radius 5. DABCF is a straight line, with D and F on the circles. Lines DE and FG are tangent to circle C and intersect at G. Find the length of FG. 15.11 B 3! [4] A swimming pool is 30m wide, 50m long end 7m deep. After an earthquake, the pool is tilted along one edge (AB in figure), the water completely covers side ABCD, but no water is lost. At this point, % of the base is covered by water. What was the water level before the earthquake? 50 20, || Nine large pipes will drain a pond in eighthours and six small pipes will drain the same pond in sixteen hours. How long will it take three large pipes and five small pipes to drain the pond? 1100 sailors found themselves marooned cn an island after their ship sank. The leader of the islanders captured them and threatened to imprison all of them the next day. However, he was willing to give them a ctance. He said that he had 100 hats, which wereeither black or white. He would place his hats on the 100 heads. No sailor would be able to see the hat on his head, but he would be able to see all the other hats. That night, the sailors worked out a strategy: they decided that each of them would count the number of black hats on the heads of all, except the first one to be called to answer. This way, either 99 or 100 sailorswould be set free. Explain why. Prove that if 0 is an interior point of triangle ABC and © AOD, BCE and COF are Cevians, then Ma 1 > & SRls (2) 2O_AF AE 80_8F 8D cO_@ ce OD FB EC’ OF FA DC’ OF DB EA By a ‘An even perfect number N must be of theform N=2'(2'"—1), where 2" —Lisa prime number. Prove that r is always even and that every even perfect number of order r > 1is the sum of the cubes of the first 2? odd numbers. “ me ABEL Let ABC be atriangle, Prove that cot +t 2+ cot =cot corZeot& ee ert a lo le 6 LESSON 1: PRIME NUMBERS AND FACTORISATION 1.1 FACTOR: The natural numbers are the numbers 1,2, 3,4, m. ‘The integers are the natural numbers together withO and the negative integers. That is, the integers are 13,-2,-1,0,1,2, 3) This lesson deals mainly with natural numbers and, sometimes, integers. The study of integers is called number theory. The sum and product of integers (or natural numbers, for that matter), is an integer (natural number). You will agree that 4 is a factor of 8, 6 is a factor of 18, 7 is a factor of 35, and so on. Is -7 a factor of 35? So, what do we really mean by the expression “a is # factor of b”?. 1.1.4 Definition of a factor ‘One way of answering this Leta and b be integers, where 00, Then ais afactor of bif is an integer, The above is an example of a definition. So, to answer the question, “Is - 7 a factor of 352”, we need to. check whether x is an integer. If the answer is “Yes”, we can conclude that - 7 is a factor of 35. Of course -5 is an integer so, we can conclude that — 7 a factor of 35. We could say that a factor is a positive integer (which is exactly the same as a natural number), if we so wish, b b If we give a name to ~, cal it n for example, we can say “=m, where mis an integer (Or we can take it further: ‘ais a factor of bit = an where nis an integer.” While all these can serve as definitions for the statement “a is a factor of b”, the last one is the one that is used most. For example, the positive factors of 15 are 1, 3, 5 and 15. However the answer to “Find all the factors of 15" is 1, -1, 3, -3, 5,-5, 15, -15. 1.1.2 Multiples ‘The multiples of 2 are called the even numbers. They have the form 2k where k is an integer. So the even numbers are: -2,0,2,4,6, 8. 6 where the even numbers corresponding to k = -3, -2,-1,0, 1, 2,3, 4,... are indicated. ‘The multiples of 3 have the form 3k, where k is an integer. They are: 3,0, 3,6, 1.1.3 Multiples and product of strings of consecutive numbers Hf we plot the multiples of 2 on the number line we get the following: To jump over only the even numbers, a frog sitting cn 4 will have to miss one number, namely, ~ 3, to get to—2, miss another to get to 0, and so on. So if we pick any two consecutive integers, one of them must be a multiple of 2. And therefore their produtt is also a multiple of 2. We have shown: For any integer k, K(k+1)is a multiple of 2, that is, tis even. Now look at the multiples of 3. Our frog sitting at ~ 6 has two jump over two numbers to get to ~ 3 and again, another two numbers to get to 0, and so on. So, if we pick any three consecutive integers, one of them must be a multiple of 3. That is: For any integer k, k{k-+1)(k+2)is a multiple of 3. It s also a multiple of 2, hence it is actually a multiple of 6. For more on this topic go to 5.4. 4.1.4 The square of an odd number is always one more than a multiple of 8 Let m be any odd number. It is one more than an even number, so it has the form n=2k-+1 for some integer k. Then n? =(2k-+1) =4k 44k +1=4k(k-+1)+1. But k(k +1) is even, which means that 4k(k +1) is a multiple of 8. This proves that the square of an odd number is one more than a multiple of 8. 1.15 Factor Properties Instead of using the cumbersome “ais a factor of b” we often write alb to mean the same thing. Ilence 4[8, 6| 18, -7|35 etc. we also sometimes say “a divices b.” Factors or divisors have some very nice and useful sroperties. Some of them a (a) 1b for any integer b. (b) _blb for any integer b. (c)_ tfalb and ble, then alc. (4) tfalb and alcthen al(b +c) and al(b ~c). (e) Always, alab. uv Learn these properties by reading them out in sentences and understanding them. 1.2 EXAMPLES 1. Write down all the positive factors of 12. Then write down all the factors. Solution 12=1.12=2.6=3.4. The positive factors a'e 1, 12, 2, 6,3, 4. The factors are 1, -1, 12, -12, 2,-2, 6,-6,3, 3,4,-4. 2. Write down all the positive factors of 60, Solution: 60 =1.60 = 2.30 = 3.20 = 4.15 =5.12= 6.10 The factors are 1, 60, 2, 30, 3, 20, 4, 15, 5, 12, 6, 10. There are 12 of them. 35 3. Determine allintegers m that have the property that ———_ isan integer. 7 35, Solution: Notice that the statement “isan integer” is exactly the same as the statement “n a ~4isa factor of 35”. So n—4 is one of the numbers 1, -1, 5, -5,7,-7, 35, -35, from which we deduce that n is oneof the numbers S, 3, 9, -1, 11, -3, 39, -31. There are eight solutions for n, 1.3 PRIME NUMBERS 1.3.1 A prime number is a number greater than 1 that has exactly two positive factors, namely, 1 and itself. Or, put in another way, a prime number is one that is greater than 1 and cannot be factorised any further. One of the most important properties of prime numbers is the following: 1.3.2 Any natural number can be written as a product of prime numbers, that is, ifn is any natural number, we can find prime numbers p, q,... and natural numbers a, b,.., and there is only one expression such that n=pta’ 1.3.3 A number which is not a prime number is called a composite number. Study the examples below carefully. They inform you on how to factorise numters into products of prime powers: 120 = 12.10 = 3.4.10 =3.2.2.2.522.22.35=2.35 3600 = 36,100 = 4.9.10.10 = 2.2.3.3.2.5.2.5 = 2.2.2.23.3.5.5 =2°35', 3600 = 60.60 = 6.10.6.10 = 2.3.2.5.2.3.2.5 = 2°3°5* 1000 = 10° = 2°.5* 8 2013 = 3. 671= 3.11.61 2014 = 2.1007 = 2. 19.53 2015 = 5.403 = 5.13.31 1.3.4 Every natural number m can be written in the form 2d where dis the largest odd factor ofn Proof: Let n be a natural number. Factorise n as a product of prime numbers. Then n= 2"d_(m will be zero, if nis odd, where d is a product of prime numters greater or equal to 3, and hence odd. Clearly dis the largest odd factor of n. 1.355 Let us try to factorise the number 317. If we divide 317 by 2, 3, 4, 5, 6, 7, 8, ... 316, we see that none of these numbers is a factor of 317. So we can conclude that 317 is a prime number, Clearly these 315 divisions can take a very long timel Fortunately, we do not have to work so hard. We need only try to divide by the prime numbers less than 317. That means we divide by 2, 3, 5, 7, 11,.. More than that, we need only go up to V317, which is just smaller than 18. So we need only check whether the seven primes 2, 3,5, 7, 11, 13, 17 are factors of 317, ‘Summarising: 1.3.6: Anumber nisa number if none of primes less than (or equal to) v/>n is a factor of n. 14 TINCT. FACTO) Let n be the number under investigation. How many distinct factors does m have? Firstly suppose n is prime number. Then it has two factors, namely 1 and n. Next, suppose n is a power of a prime number. For example, suppose n= 3*=81. Then its factors are 1, 81, 3, 27, 9. They a’e all powers of 3, namely 1, 3, 3%, 3%, 3¢. So 3*has 5 factors. If 3 is replaced by another prime number, say 5, we will again obtain 5 factors, namely, 1, 5, 5?, 5? and 5* It should be now clear that 1.4.1 For a prime number p, the factors of p” are 1, p, p*, p, p.., p". So p™ has m +1 factors. Now, any natural number can be written (uniquely) 3s a product of powers of primes, as we have already seen. 6 x 25 = 24", How does one count the number of its factors? Example: For example, 400 = 4 x 100 ‘One way is to go through the factors in pairs: 1, 400, 2, 200, 4, 100, 5, 80, 8, 50, 10, 40, 16, 25, 20 giving 15 factors altogether. 19 Well fixa factor of 5 say, 5. There are five factors of 400 associated with 5, namely, 115), 205), 25), 245), 245) This can be done for each factor of the three factor: of 5#, namely 1, 5 and 25. They are 1 2 2 Bo 1S 250 BS BSS 1s? 25? 285? 225% 2t5t So again we come to 15 factors. But observe that 15 arises from 15 = 3.5. More precisely, for each of the five factors 1,2,2°,2" & 2" of 2° three new factors may be obtained, by multiplying each of these five with the three factors of 5. ‘Summarising: the number of factors of 25" is (4#1(2+1) = 5.3 = 15. This observation allows one to calculate the number of prime factors of any natural number quickly: 1. Factorise the number into prime powers. 2. If pis one af the prime powers in the factorisation, then pf has n 41 factars, as we have seen, 3. Add 1 to each of the powers of the prime fa:tors and multiply all of them. The answer you get is the number of factors of the number that was given. 1.4.2 f n=p,°p,'p,..... is the factorisation of a into prime powers, then n has (a+ 1) (b+ 1)(c +1) .. factors altogether. 1.3.33 If mis square number, say m=n'*, then m=(p"q°...)’ =p"q"... hence the powers 2a, 2b, .. in the factorisation of m into prime powers are all even .. Likewise if m is a cube, the powers in the factorisation of m into prime powers are all multiples of 3. Examples: 1. We saw earlier that 400 has 15 factors and 60 has 12 factors. This is because 400 = 245? and (4+1)(2+4) =15. Similarly, 10 = 2.3.2.5 = 273.5 and (2+ 1)(1 + 1)(1 +1) =12. How many factors does 12 800 have? Answer: 12 800 = 2°.5%, so it has (9 + 1), (2+ 1) = 30 actors. 1.5 DIVISIBILITY RULES The following rules provide quick and easy ways of ceciding whether one of the numbers from 2, 3, 4, 5 6, 8, 9 or 11 isa factor of a given number. We state them now and reserve their proofs for Lesson 3.6. Anumber is divisible by + 2ifthe last digit is even ‘© 3ifand only ifthe sum of its digits is divisible by 3. ‘+ 4ifand only ifthe number formed by its las: two digits is divisible by 4. * Sif the last digit is either 0 or 5. + 6 ifitis divisible by 2 and 3. «8 if and only ifthe number formed by its las: three digits is divisible by 8 © 9ifand only ifthe sum of its digits is divisible by 9 ‘© 11 if and only if the difference between the “alternate sums” is divisible by 11 EXAMPLES 1 1.1 43 527 684 is divisible by 2 since 4 (last digit) is even. 1.2 56 178 951 is divisible by 3 since +6+14748+945+ 1.3 34 519 068 is divisible by 4 since 6 1.4 9 745 875 ends in 5 and so is divisible by 5. 1.5 86 594 384 is divisible by 8 since 384 = 8.48 s. 1.6 675 483 264 Is divisible by 9 since 6+745#44843+2+644 = 45 is divisible by 9. 1.7 358 296 004 is divisible by 11 since (3+8+940+4)-(5+2+640) = 11 is divisible by 11, 42 is divisible by 3. 2. What is the remainder when 45 190 861 is dvided by 4? ‘Answer: Divide the last two digits by 4. 61 dvided by 4 gives the quotient 15 and remainder 1. Therefore 45 190 861 leaves a remainder of1 when divided by 4. (Another way of looking at thi 45 190 860 is divisible by four so the remainder is 45 190 861-45 190 860 = 1. 1.6 RATIOS AND AVERAGES 1.6.1 When one quantity (x) is divided by another (y) , the answer is often referred to as the ratio of, the one quantity to the other. Thus the ratio of the first quantity to the second is ~ , we also wntte the quotient as x: y y Example: 65% of the children at a school are girs. What is the ratio of boys to girls in this school? Answer: If 65% are girls, then 35% are boys. So the ratio of boys to girls is 35:65: 2 Zz 1.6.2 Averages: The average or mean of a set of numbers is obtzined by adding all the numbers in the set, and then ing by the size of the set. ‘Thus, if we have a collection of n numbers in a set, and the sum of the all the numbers is S, then the s average Ais given by A=—. n Another way of saying the same thing is: If $= sum of numbers in a collection = number of numbers in the collection ‘A= average, then S=nA. 1.7 RATIONAL AND IRRA’ NAL NUMBERS .. are called rational numbers. m So rational numbers are numbers that can be writte1 in the form —, where m and n are integers. a Numbers that cannot be written in this form are called irrational numbers. The positive rational numbers are also called fra Consider the numbers that are between 0 and 1. Think of them as being dots located on the number line. 1 If we divide this ine segment into two equal parts, we obtain a dot atthe number = lf we proceed in this manner, that is, divide the segment into n parts for all possible natural numbers n, then the segment is populated with infinitely many dots, all of which are rational numbers, and the dots account for all fractions between 0 and 1, A question now presents itself: are there numbers between 0 and 1, which are not amengst the dots obtained? This question is exactly the same as: Are all the numbers between 0 and 1 rational numbers? For a long time, it was held that the number line is made up entirely of rational numbers and nothing else. The bomb fell when through a simple logical argument, it was conclusively proved that the number whose square Is equal to 2is nota rational numbel. Which meant that amongst the points on the number line were these dark objects which could nct be obtained by dividing one integer by another, the 0 called irrational numbers. A fraction is said to be in lowest terms if the numerctor and denominator have no common factors other than 1, that is, if they are coprime. By removing common factors from the numerator and denominator, 189 any fraction can be reduced to lowest terms. For eample, 95 = = nal number 1.7.1 V2 is an ier Proof: We prove by contradiction. Suppose there i: rational number whose square is equal to 2. That m is, there are integers m and n such that /2="" . a We can assume that m and n have no factors in common. ‘Then m' =2n". So m*is even. This forces m to be even. (If m is odd, then so is m’). So2isa factor of m. Let us write m as m= 2k where kis an integer. Substitute into mm =2n" to get (2k)* =2n’ . Simplifying we get 1? =2k°. This means that 2 isa factor of n®, :0, as before, is a factor of n. Now we have that 2is 2 factor of both m and n. But we said in the beginnirg that m and n have no common factors! This is the contradiction that we were looking for. We have proved that the square root of 2 is an irrational number. m where “is in lowest terms. This would mean that of ” n all the ways that /2 can be written as a fraction, n will be the smallest amongst the denominators. Now, Alternate Proof: Again let us assume that V2 = since 1<2 <4, we must have 1 < V2<2, that is 1< "<2, son0 (given) and z< 100, from equation (1). So2= 80, 80y = 19.80, y=19, xX +19 + 80 = 100, from (1). x=, ‘There is only one solution: (x,y, z) = (1, 19, 80) 2.7 INTEGRAL SOLUTIONS TO THE EQUATION ax + by =c In this section we try to find all pairs (x, y), both numbers being integers, that satis ox + by=e. fy the linear equation Examples: 1. Find all integers x and y such that 2x + 2y Solution: If (a,b) is solution then 2a + 2b=5, and a and b are integers. So 2(a + b) = 5. But ‘a+ bisan integer. So the left hand side is even, while the right hand side is odd, This is impossible {no number is even and odd at the same tirre). The conclusion is that no solution exists. 2. Find all integers x and y such that x + 2y=5 Solution; x = 5~2y is an integer whenever ys an integer. So the solution setis {(5~2y,)} where y Is an arbitrary integer. We usually write this as: Solution Set = {(x,y)=(5-2y,y)y isan integer} for, we can use the symbol Zwhere Z= 4..,-2,—1, 0,1, 2,.«} is the set of integers and write: Solution Set= {(x7)=(5-2y. yy eZ} The vertical sign | is read as “such that”. Sowe read the above as: the set of all (5 ~ 2y, y) such that y is an integer. For example, ify = 200, then x= 5 ~ 400 = -395 and (-395) + 2(200) = 5 so (x, y) =(-395, 200) is a solution. There are infinitely many solutionssince we can substitute infinitely many numbers for y Determine the solution set of 4x + 11y=8, where x and y are both integers. Solution: We can rewrite the equation as Lly = 4(2~x). Since 11 and 4 are coprime we can conclude (see 2.4.1 above) that 11 is a facter of 2 - x. So 2—x = 11n for some integer n. Sol for x, we get x=2-1n. Substituting, 4(2 ~ 11n) + 11y = 8. Now solv2 for y: Ly=8-8+44n, y=4n. $0 (x, y) = (2-11, 4n) where nis any integer. Check: 4(2~141n) + 11(4n)= 8~44n + 44n= 8 for any value of nso (x, y) = (2 11n, An) is indeed a solution for every integer n. Solution Set = {(x,y)=(2—11n,4n) | neZ} Determine the solution set of 67x + 35y = 113, where x and y are both integers. Solution: Clearly, this is not at all obvious. Te methods used in the previous examples do not work here. We proceed as follows. Step 4: Find at least one solution Solve for y: (or x if you prefer) 35 (see Lesson 3: 3.1 & 3.2 on the “division algorith That is, write each in the form 35q + r. 113-67 _ 3(35)+8-(35.1+32)x 35 35 8-3 35 aot 8-32x ‘Step 3: Find an integer such that isan integer. or any integer x, the right hand side is an irteger provided 35 is a factor of 8 — 32x = -8(4x-- 1). Since 35 and - 8 are coprime, this happens when 35 is a factor of 4x1. Now 35n = 4x—1 immediately gives a solution for n= 1, namely x=9. and then 832K ys 3ox+ =9+8(-1)=-14. x=9 and y =~ 14 isa solution of the equation 67x+35y = 113. Step 4: Let (x, y) be any solution. So we have: 67+ 35y = 113...(1) 67(9}+35(-14) = 113...(2) Subtract: 67(x—9) +35(y+14) =0 67(x-9) =-35(y +14) Now 67 and ~ 35 are coprime. Hence 67 is afactor of y # 14. That is y+ 14 = 67n for some integer n Soy=67n—14. Substitute: 67+ 35( 670 ~ 14)= 113 67+ 67(35n) = 113 + 35.15 = 113 + 490 = 693 = 67.950 x+35n=9, Thatis x= 9-35n. So (x, y) = (9 ~35n, 677 14), where n is an arbitrary integer, is a solution. For example, if we let n = -12, we obtain the solution x= 9~ 35 (- 12) = 429 and y = 67(-12) - 14 = - 818, This can be checked by substitution: 67(429)+ 35(-818) = 28743 ~ 28630 = 113, Solution set is {(x,y)=(9-35n, 67n—14) | (You can test this answer. Substitute n = 10 ‘or example and check that (-341, 656) is indeed a solution of the equation 67x-+35 y= 113), 2.8 APPLICATIONS OF LESSON 2 BL 133 134 136 158 | 221, 283 293 LESSON 3: CONGRUENCE ARITHMETIC 3.1 THE DIVISION ALGORITHM ‘An integer can be divided by another, called the divisor, to produce a quotient and a remainder. For ‘example, when 65 is divided by 11 (the divisor), the quotient is 5 and the remainder 10. Note that quotient remainder + + LL + 10 ‘When carrying out this process we found the highest multiple of 11 that is Jess than 65. This ensures that the remainder is a positive (perhaps zero) number less than 11. If we started with a negative say ~ 65, we can still obtain a positive (actually non-negative, as it could be zero) remainder less than 11; the largest multiple of 11 less than 65 is -66 so 65 6). +1 The Euclidean Property : Let m be eny integer and n a natural number. Then m can be divided by n to obtain a quctient q and remainder r. More precisely, m=qn+r where 0 x=y even if a is not zero. 1 Also when dealing with congruences, there is no such thing as —. However, we may, under a certain conditions, make the conclusior (ax =ay => x=y)), without “dividing both sides by Examples 1. Let us confirm some of the rules above for n= 8, (So m of 8) whenever m~nis a multiple We have 19 =,(-5) and 15 (19 + 15)-(-5 455) =-16 (29-15) - (5-55 (19.5)~ (5.55 198 (65) are all multiples of 8. From definition 3.4.1, 5 as can te easily checked. (19+15) = 2 (-5+55) (19-15) =a (5-55) (19.15) =0(-5.55) 1s=s (57 Illustrating the truth of the first four assertions of 3.5.1. 2. Show that 4x=, 4y=>x=, y. Proof 1: 4x=, 4y>9|4x—4y 9 /4(x—y) Now in 2.4.1 we said: If a|bc and (a,b) = 1, then alc. Since 9| 4(x-y) and (9,4) = 1, we can therefore conclude that 9](x-y). That is x=, y. Proof 4x =, 4y = 714x) =, 7l4y) =2 28x =, 28y => x=, y, where we have used 28, 1 3.6 PROOFS OF DIVISIBILITY RULES In Lesson 1.5, we stated a number of “divisibility rules”. Congruences provide a powerful tool to prove these rules. This is what we shall do in this paragrapy 3.6.1 Divisibility by 2 and 5 Let us use the symbol n for an arbitrary natural number and m for the last digit of n. Then n—misa multiple of 10, and hence a multiple of 2 as well as §. That is: Hf mis the last digit of n, then n=2m and n=sm. Hence © ifm that is if the last digit of n is divisible by 2, so is n. (see 3.4.2) , then '=20; that is if the last digit of nis 0 or S, then mis a multiple of 5. 3.6.2 Divisibility by 4 and 25 Let us use the symbol n for an arbitrary natural number and m for the two-digit number that make up its last two digits. Then nm is a multiple of 100, and hence a multiple of 4 as well as 25. That is: n=, mand n=,m. So n and m leave the same remainder when divided by 4, or by 25. Hence if m (See 3.4.2) 0, then n=, 0; that is, if m (the 2-digt number at the end of n) is a multiple of 4, so is n And if mz;,0 then n=, 0; thatis, ifm isa multiple of 25 (that is if the last two digits are 00, 25, 50 or 75), then isa multiple of 25. The rules for divisibility by 8 and 125 follow the same lines. 3.6.3 Divisibility by 3 and 9 For any natural number k, 10" ~1is a number havirg only 9's and hence is a multiple of 9, and (hence) a multiple of 3. 10" =,1 (1) and 10° =, 2) Now let n be any natural number. Let also: abe its unit digit 4a, be its unit digit 4a, be its unit digit a, be its unit digit, and so on. Then =a, +100, +10? +109, to. which, from equation (1) and repeated use of 3.5.1,is congruent (mod 9) to +0, +0, +0, +... Thatis 1, (0, +0, +0, +0, +o). Hence n and 0, +a, +a, +a, +... leave the same remainder when divided by 9. 50, if (a, +0, +4, +0, +...) 5 Othen ns, 0. Th s of nis 2 multiple of 9, then so isn ifthe sum of the ‘The same argument can be applied to 3, to conclude that if the sum of the digits of nis a multiple of 3, then ois. 3.6.4 Example: The number n = 7777...777 has 201 sevens. Find theremainder when n is divided by fa) 4 (b) 25 () 9 {d) 3 10 the remainder is 1. (b) n=, 7,2 so the remainder is 2 (0) 05 (74+747...+7)5; 201.7), 14075, (1+4+7)=, 3. The remainder is 3. (4) =, (74747 +..+7)=, 1407 = Osince 1407 = 3.169, The remainder is 0, that is n is a multiple of 3. 3.6.5 Divisibility by 11 Note the following: 10" =100" 10" 5,105, -1 Hence: 3.6.6. An even power of 10 is congruent to 1 mod 11 and an odd power of 10 is congruent to -Lmod 11, This observation allows us to conclude the following Let N=4,4,_ 4, 5.--@y0, be an (n + 1) digit number. Then N= Gy yall hly a, +10a, +1002, +1000, + Which is the same as: N= dy + 10a, +1004, +1000, 05, (Cy $y iy to) Uy Hy Hy +o) ‘The right hand side is the difference between “alternate sums" of the digits of the given number. Hence: +a, ~a,+ 3.6.7 6,4 dy sendy Ghly and (a, +4, +4, + leave the same = (a, 44, 4,4 remainder when divided by 11. In particular, if the remainder is zero, we have 04, dy, yo, Is divisible by 11 if and only iflay +4, +4, +...) (G+ +s +o) divisible by 11. 3.6.8 Examples Use congruences to decide whether 47680245 is (a) divisible by 9 (b) divisible by 11. Solution (a) 4+7+6+8+0+2+4+5=36 =, 0) so the emainder is 0. So 47 680 245 is divisible by 9. (b) (4+6+0+4)~(7+8+2+5)=-8 which is not divisible by 11. So 47 680 245 is not divisible by 11. Check! 3.7 PARITY. The even integers are 6, -4, -2, 0,2, 4, 6, The odd integers are 5,-3,-1, 1,3, 5, Parity is the property of an integer to be even or oad. So two Integers have the same parity if both are even or both are odd. It immediately follows that the ‘sum of two integers having the same parity is always even. From this it immediately follows that if the ‘sum of two integers is odd, then they have opposite parity. This rule does not apply to products; the product of odd integers is odd and the product of even integers is even. These observations lead us to newer ones; The sum of any set of integers, all of which are even, is even. The sum of an even number of integers, all of which are odd, is even. The sum of an odd number of integers, all of whichare odd, is odd. I the sum of a set of integers is even, then there isan even number of odd integers in the sum. If the sum of a set of integers is odd, then there is en odd number of odd integers in the sum. If the product of a set of integers is odd, then all th numbers in the set are odd. If the product of a set of integers is even, then at least one of the numbers in the set is even. For example, itis impossible to find 5 odd numbers that add up to 100, because the sum of every 5 odd numbers is odd. Itis very useful to remember the fact that a number is even when it is congruent to 0 mod 2 and odd when it is congruent to 1 mod 2 and also that 1+1 =, 0. 62 6 a 68 8 70 7h 72 3B 78 79 [85 142 144, 14s | 146 186 225 230 | 234 256 | 259 281, 307, LESSON 4: SOME TIPS ON COUNTING 4.1 THE PRODUCT RULES IN COUNTING Suppose A, B and Care three towns. If there are three roads connecting A and B, and four roads connecting B and C, how many different ways are there to get from A to C? 2 : - Oo=-O-=—@ 3 Let us name the roads from A to B by 1, 2 and 3, while the roads from B to C are named a, b, cand. By 1a, we shall mean the path from A to C, using road 1 followed by road a. If we choose road 1, there are four possible routes from A to C, namely, 1a, 1b, 1cand 1d. Likewise, with road 2, we have 2a, 2b, 2c and. 2d. And with road three, we have 3a, 3b, 3c and 3d. Ans: There are 12 ways to get from Ato C. We could have arrived at 12 using the following argument. There are 3 ways of choosing the first road. For each choice of the first road, there are 4 choices for the second road. Hence there are 3 x 4 = 12 ways of choosing both roads together. We can generalise: 4.1.1 THE FIRST PRODUCT RULE: Suppose there are m ways of doing one thing, and n ways of doing another, after the first has been done. Then - assuming no other restrictions - there are mn ways of doing both things together. 4.1.2 Examples: 1. How many two-letter “words” can be made using the letters of the word CAT? Method 1: We can actually make the words First letter C: CC, CA, CT First letter A: AC, AA, AT First letter T: TC, TA, TT Answer: 9 words Method 2 (preferred): The first letter can be chosen in 3 ways. (Itis one of C, Aor T). For each choice of the first letter, there are three choces for the second letter. So there are 3 x3 = 9 ways of choosing both letters together, that is, of making two lettered words. 2. How many three- letter “words” can be made using the letters of the word CAT? ‘Answer: For each choice of the first two letters, there are three for the third. For example, if the first two were chosen as AT, then we can form ATC, ATA and ATT. But there are 9 ways of choosing the first two letters. So there are 9x 3 or 3x3 x3 = 27 words that can be made, ‘The above example gives us a method of soWing more complex problems. 3, How many 4 lettered words can be made from the word MASTER? ‘Answer: The first letter can be chosen in 6 ways, the first two in 6 x 6 ways, the first three in 6 x6 x 6 ways, so the four letters can be chosen in 6 x 6 x 6 x 6 = 1296 ways. So there are 1296 such words. We have the following rule: 4.1.2 THE SECOND PRODUCT RULE IN COUNTING : Suppose we need to filln r boxes. If the first box can be filled in m, ways, the second in m, ways, third in m, ways, and so on, until the r* box, which can be filled in m, ways, then all r boxes can be filled in myn, 0, WAYS. m, | m, ™, 4. How many two lettered words can be made from the letters of the word CAT, ifno letter is to be repeated? ‘This means that “words”: like CC are not allewed, ‘The words are: CA, CT, AC, AT, TA, TC. There are six such words. r, better still: ‘The first letter can be chosen in three ways, but the second in only two ways, since the first cannot be used. Using boxes: 3]2 which leads to 3x 2= 6. 5. How many 5-letter words can be made ising the letters from A to G only, if no letter is to be repeated? if repetitions are allowed? Answer: We need to fillin 5 boxes, ‘The letters are to be chosen from the sevenetters A, B, C, D, E, F and G. The first letter can be chusen in 7 ways, Ue second in 6 and so un We haves 71 6[5]4]3 So there are 7x 6x5 x 4x 3 = 2520 words. If repetitions are allowed, we have 77171717 Answer: 7° 70 4.2 ARRANGEMENTS OR PERMUTATIONS 4.2.1 Definition: Let n be a natural rumber. The product of the first n natural numbers , that is , the product 1.2.3.4..n, is written as nf. That isn! =1.2.3....n, (Read nl as “n factorial”.) We also define 0! by simply saying: 0! = 1. 4.2.2 Example Show that the number of 5-letter “words”, with no letters repeated, that can be made {from using the letters of the word STRANGE is 3 Proof: We need to fill five boxes with no reretitions (see example (5) above): 8]7[6[5]4 So the number of words is 8.7.6.5.4. Now a 3 d, yr as requies 4.3 COUNTING ARRANGEMENTS Generalise (6) above: Show that the number of r- letter “words” that can made from a word having n different letters, repetitions not allowed, is a! (n-n" Proof: We have to fill r boxes, without repetitions: ni} nt | n-2| m3] .... | mertd nln -1)(n-2)..(n=r +1) = (The second box is n-1, third is n-2, ., $0 the r" box is n—{r-1) = nr + 1), Hence the number of such words is ata 1)n—-2)...da—r +1)=(n=r +1)(—F +2)... 1.2.3.....(n=(n=F+1)...(9=1)n 123...(0=n) nl (n-n)! since the numerator is just the product of tre first n natural numbers. n Another way of stating the above result, without resorting to “words” is: 4.2.2 The number of ways of arrany time is ig n objects taken rat a nt (n-H)! ‘The number is sometimes written as “P., which reads as “the n(n—1)(n—2)..(n—r-+1) number of permutations of n objects, taken rat a time” 4.4 APPLICATIONS OF LESSON 4 34 99 121 122 123 | 424 125 216 299) n LESSON 5: BINOMIAL COEFFICIENTS 5.1 NUMBERS OF THE FORM at Reflect on the following problems: 1 Find a formula for1+2+3+4....+n In how many ways can two objects be chosen from a set having n objects? You have n points on a flat surface, no three of which lie on the same straight line. A straight line is drawn through every pair of these points. How many such lines are there? 4. n points all on the same straight line. A point P that is not on the line is joined to all n points. How many triangles are so formed? 5. To win a competition, you have to pick the frst two horses in a race. What is the least number of picks you have to make to be sure of winning, if n horses have entered the race? 6. Whatis the coefficient of x° in the expansicn of (1+x)"? Problem 3: Find a formula for 142+3+44--+n When schoolboy named Gauss was given this protlem, with m= 100, he solved the problem in no time, using an ingenious method . We shall use the samemethod here. By the way, Gauss went to became the one of the greatest scientists who ever lived. Let x be the unknown sum. Gauss recognized that te value of x does not change if the sum is written backwards. So 424 3 bean +(n-1)+(n—2)+--+1 ‘Add both sides: 2x=(n+1)+(n+1)+--+(n-4+4) and there are n terms in this sum. So 2x=n(n+1) nln+4) “2 We have shown: 5.1.2 The sum of the first n natural numbers is aln+1) 2 142434) B Problem 2: In how many ways can two objects be chosen from a set having n objects? Let us number the n objects; so we denote the set of objects as {1, 2, 3, 4,.., n}. To list all the pairs of two objects, we can start by insisting that 1 is in the set, then by insisting 2 is in the set, and so on. In this way wwe can list all pairs as follows: (2,2), 4,3}, 1, 4}. m} -uithere are n= 1of them (2,3), (2, 4), (2, 5),...(2, nm). {3,4}, {3, 5},{3, 6}.....{3, m} . there are n~2 of them there are n-3 of them {n, 1-1}. (there isonly one). Note that all pairs have been counted. And we have 14+2+34+4+---+(n—1) of them, So, two objects can be selected from a set having nobjects in 1+2+3+4-+--+(n— ways. (Here we used the result from problem 1.) Problem 3: You have n points on a flat surface, no three of which lie on the same straight line. A straight line is drawn through every pair of these points. How many such lines are there? Any two points determine exactly one line, and every such line is associated with exactly one pair of points. So the number of lines is the same as the number of ways two points may be chosen from n nln=1) points. We have seen in problem 3 that this numbers, Problem 4: 1 points all lie on the same straight line A point P that is not on the line is joined to all n points. How many triangles are so formed? Three vertices determine a unique triangle; so the number of triangles is equal to the number of ways three points may be chosen. One of them is always P. So we have to choose two points from the n points nin-1) on the line, to create a triangle. The number of ways that this can be done is from Problem 1. ™ Problem 5 : To win a competition, you have to pick -he first two horses in a race. What is the least number of picks you have to make to be sure of winring, ifn horses have entered the race? This should now be clear: there are oe 2) ways otpickng the two horses 5.1.3 Definition: Let n be a natural number. That is, n(1,2,3,4,....} .A number of the moet is called a triangular number. form Problem 6: What is the coefficient of x’ in the expansion of (1+x)"? Consider instead the product (1+x,)(1+%,)(1+%}(1-+%,) . Amongst the terms in its expansion are those which are products of two of the x's like XX, %:Xs, XeX3and s0 on. How many such products are there? It will be as many ways as we can choose two objects from the set 1% 1% Kaye), Which has n elements. We have seen in problem 2, that this numbers !—4? , Now make all the x’s equal to x. That is, let KEK, HX =X % a 2 nln-1) Then the original product is (1+)", all the productsof pairs are equal to x’,and there are of nln) them, so the product of pairs sums to X°, The coefficient of xin the expansion of (L+x)"is, n=) al therefore 5.2 COUNTING SUBSETS Reflect on the following problems: 1. Aset has five elements. How many subsets, each having 3 elements, does it have? How many five-element subsets does a set having 10 elements have? 3. Youneed to select four vertices of an octagon (an &-sided polygon). In how many ways can this be done? 4, Towin the Lotto, you need to select 6 numbers from 48 numbers. In how many ways can this bbe done? What are the chances of winnng the Lotto? 5. How many subsets does a set of n elements have? 5.2.4 Problem1 5.2.2 We listall the 3-element subsets: {a;b; ¢} fa; bjd} fa bie} fad} facie} fadje} {b¢; d} {b; ce} {b; de} 5 {e:d;e}. ‘The answer is 10. 5.2.2 Akey observation Note that we did not write, for example, {a; ¢; b; since this set is the same as {a; b; c). In this problem, the order of the symbols is not important. But the word abc is different from the word acb. Indeed, 3! words can be made from (a; b; ch if repetitions are not allowed. Each of the 10 three- element sets above, gives rise to 3! words, so the total number of three-letter words that can be made from the five given letters is 10(31) = 60. But we also saw, in Examples 4.1.2(5) and 4.2.2(1), that the total number of three-letter words thatcan be made from five letters is 5.4.3 =60. So 10(3!) 4.3, nerermenel This observation, namely, that there are two ways of counting the number of “r-letter words” that can be made from a set having m letters, can be ised to count the number of r-element subsets of a set having n elements. The argument above can be used to count the number of subsets having any number of elements! 5.2.3 Problem 2 : How mony 5-element subset: does a set having 10 elements have? Answer: Let N be the number of such subsets. from the previous example, N(S!) = 10.9.8.7.6. 10.98.76 _109.8.7.6 So the number N of 5-element subsets is = 5 12345 which simplifies to 252. In general, the problem is to determine the number of r-element subsets a set having n elements thas. The above example provides the answer. 5.2.4 (Number of subsets of a given size) Given a set having n elements, the number of subsets having r elements is n(n—1\n r+) 123.0 . (The denominator is the product increasng from 1 to r, and the numerator is the product beginning with n and decreases ending when r factors are exhausted). n This number is written as °C, or (") 6 5.3 SOLUTIONS TO PROBLEMS 3 AND 4 Problem 3: You need to select four vertices of an octagon (an 8-sided polygon). In how many ways can this be done? Since the order is not important, the count here is the same as the number of subsets, each having four elements, that an 8-element set has. Answer: S265, 70 ways Problem 4: To win the Lotto, you need to select 6 numbers from 48 numbers. In how many ways can this done? What are the chances of winning the Lotto? ‘Answer: Again, the number of ways is the same as tie number of 6-element subsets a set having 48 elements has, which is 48.47.46.45.44.43 123456 12271512. The chance of winning the Lotto is less than 1 in 12million! 5.4 BINOMIAL COEFFICIENTS Let us look again at the Lotto problem. We determired the number of ways of selecting 6 numbers from 48. Is it possible to write the answer using only 6 and 48? The answer is yes: The number of ways is 48.47.46.45.44.43 _ 48.47.46.45.44.43 123456 él _ (48.47.46.45.44,43)421 ~ 6142! ___ 48! 61(48—6)! In general we have 5.4.1 Given a set having n elements, thenumber of subsets having r elements , nln-A)(n—2)..(n—r+1)__nl Want ri(n—r)! where ris one of 0, 1,2, ..n, is \(n—2). nia-3(n-2). al 123.06 rio! The natural number is z very important number, so important that it can be 0 found on hand-held calculators inthe form nCr (ead 38m choose r)or ( ) , 1/8) ( 5.4.2 Recall that 0! = 1 (see 4.2.1). So for a natural number n, we have (°) Ol(n-O} Lat (4 n) =a = lo -1) 2) 2 al ne i a ald 5.4.3 An expression like 1 + x is called a binomial, The first four powers of 1 +x are: (+x) =14x (Qt+x) =142x+x7 (14x) =143043x 8 (Lex) =144x+6x? +4xtx! Notice that the coefficient of x in each of the first foar powers is the same as the index; that is the coefficient of x in the expansion of (1+x)" would appear to be n. We saw earlier that the coefficient of x” 1) 5 11) so in each ofthe four cases above, we he ooar=(2)(t}eo(2 + ‘The above formula actually works for any natural number n. The formula is usually referred to as the 0 Binomial Theorem and the numbers ( re also called binomial coefficients, \r 8 5.5 THE NUMBER OF SUBSETS OF A SET Let us now turn to Problem 5. Problem 5: How many subsets does a set of n elements have? There is a clever way of counting this number. Suppose we want to count how many subsets (1; 2; 3; 4; 5; 6; 7} has. Draw seven boxes as shown: Take any subset, like (2; 4; 7). Put 1’s in columns 2, 4 and 7, and zeros elsewhere. ofijofafojofa The empty set: The subset (6} PrPeeee ey So: To count the number of subsets, we can instead count the number of strings of zeros and ones we can create in our box. But this is easy, if we use the product rule! Each box can be filled in only one of two ways, so the number of subsets, by Lesson 4.1is 22X22X2X2K2 =128 There are 2’ =128 subsets all together. Of course, there is nothing special about the numbe- 7. The same argument can be used for a set having n elements, where nis any natural number. 5.5.1 Let a set S have n elerrents. Then Shas 2" subsets. 5.6 PROBABILITY Lesson 4 and much of what we did in this lesson, answered the question “How many?” To recall: © Ifone thing can be done in m ways, and another inn ways after the first has been done, in how many ways can both things be done together? (Answer: mn) + Inhow many ways can n objects be arranged? (Answer: 1!) + Inhow many ways can r objects, taken froma objects, be arranged? (Answer: n(n—1)(n—2)...(n—r +1) In how many ways can r objects be selected from n objects, if order does not matter? (n—I){n-2)...(n—r +1) Answer; an 1234.0 ="C, ‘* How many subsets does a set of n elementshave? (Answer: 2") To calculate the probability that some event is likely to happen, one needs to divide the number of “successes” by the number of “total outcomes’, if itis at all possible to do so. The above formulae are very useful in this regard. 5.6.1 Example coin is tossed 6 times and the outcome, namely Hor T (that is, a head falls, or a tail falls), is recorded. ‘Thus HHTHTH is a possible outcome. What is the probability that the record shows three heads? Solution: Each of the six slots can be filled in two ways, so all together there are 2° = 64 possible strings. (Second counting principle, Lesson 4.1.) 654 For three H’s to occur we have to select three slots from 6, and this can be done in °C, 3 ways. 5.7 APPLICATIONS OF LESSON 5 268 LESSON 6: POLYGONS & PLATONIC SOLIDS ‘triangle is an example of a polygon. It has three sides. Four-sided polygons are called quadrilaterals, five-sided ones are called pentagons, and there are 1ames for polygons with more sides, as given below. 6.1 REGULAR POLYGONS 6.1.1 A regular polygon is a polygon having alts sides and angles equal. The first eight regular polygons, along with their names, are shown below. O00 Equilateral Square Regular Regular Regular Triangle Pentagon Hexagon Heptagon Regular Regular Regular Octagon Nonagon Decagon In general, a polygon has n sides, and n vertices where m is greater or equal to 3. In the case of a triangle, the sum of the three angles is, as you know, 180". What is the sum of the angles of a quadrilateral? Of a pentagon? Or, for that matter, any polygon having sides? Fortunately, we have a very nice formula to calculats the sum of the angles of a polygon having any number of sides, say n sides. 6.1.2 The sum of the (interior) angles of any n-sided polygon, regular or not, is (n - 2) straight angles, that is, 180(n - 2) degrees. To see why, let x be the sum of the angles of a polygon having n sides. In the figure below, n = 7. O &® Take any point P inside the polygon, and join it to the n vertics have 3 xn = 3n angles altogether. The sum of these 3n angles is 180ndegrees. ‘triangles are thereby formed. They 31 We can add the all the angles in a different way: ‘sum of the n angles at P + sum of the remaining 2n zngles. so. 180n= sum of the n angles at P + sum of the remainng 2n angles. isthe Of course, the sum of the n angles at P is 360 . Now the “sum of the remaining angles’ is just the sum of all the interior angles of the polygon (can you see this?], which we called x. That is, 180n=360+x Solve for x; x=180n-360' = 180(n—2) degrees, thet is, the sum of the (interior) angles of a polygon having m sides, is 180(n~2) degrees. Hence the sum of the interior angles of a triangle, quadrilateral, pentagon, hexagon and heptagon are 1, 2, 3, 4 and 5 straight angles, which are 180°, 360', 540°, 720° and 900’, in that order. In the case of regular polygons, the n angles are equal, so in this this case, we can calculate the size of each angle: Equilateral triangle: 180/3 = 60°, as we know. Square: 360/4 = 90", as we know. Regular pentagon: 40/5 = 108, Regular hexagon: 720/ 6 = 120 Regular octagon: (6 x 180)/8 = 135° and so on. In general the interior angle of a regular polygon having n 2 6.1.3 The sum of the exterior angles of a polygon is 360°. Note that (a + b ++...) + (sum of interior angles) = sum of m straight angles. That is (a+b+c+..) + 180( n-2)=180n. Hence (a+b+c+..) = 180n ~ 180(n-2) = 180n ~ 180n + 360 = 360". ‘An altenative approach is to imagine that you are walking around the above polygon in a clockwise direction, and return to the starting point. What is the measure of the angle through which you would 60" have turned? Clearly, itis 360 degrees. But this is alto the sum of a, b,¢,.soa+b+c+d-. 2 6.2 REGULAR POLYGONS & CIRCLES Let ABCD... be a regular polygon having n sides. Let the perpendicular bisectors of two adjacent sides, say AB and BC, meet at O, as in the diagram below. We claim that OA = 0B = OC = OD = .. and hence the circle centred at 0 having radius OA contains all the vertices of the regular polygon. Proof: Any point on the perpendicular bisector of a ine segment is equidistant from the endpoints of the line segment, (See Lesson 15.7.) Since O lies on the perpendicular bisectors of AB and AC, we have that A= 08 = OC. Hence a=bandc=d. But AB = BC sce the polygon is regular. Hence triangles OAB and ‘OBC are congruent (5, 5,5) from which we see that k= c. Since the angles of a regular polygon are also equal, b+c=d+.e, 50d =e. t follows that triangles OCB and OCD are congruent (s, Z, s}, so 0B = OC = 0D. Proceeding in this manner, we can conclude that O is ‘equidistant from all the vertices of the polygon. That is, the circle having centre O ‘and radius AB contains all the vertices of the polygon. Hence: 6.2.1 The vertices of a reguler polygon all lie on a circle. IfXY and PQ are equal chords of a circle centred at O, that is, XY = PQ, then, since OX = OY= OP = 0, triangles OXY and OPQ are congiuent (s, 5,5). Therefore ZXOY= Z POQ. That is, equal chords of : circle subtend equal angles at the centre of the cc, LI4 REA _ theorem in geemetry says thatthe angle subtended by a chord at the centre of thecircle is twice the angle subtended by that chord at any polncontheStcurrene. We ca oandiode fom ss that ama 6.2.2 Thus, if a vertex of a regular polygon is joined to all the other IN L vertices, as shown above, then all the angles fanned out are equal to each other. => Suppose a regular polygon has n sides, each of which has length a Draw the circumscribed circle, letting h be the distance from the centre to a side cf the polygon, and 20 the angle subtended by a side of the polygon at the centre of the circle. We have So the area of the polygon = n times (area of each triangle) no na Go ag tives the ara ofthe fn? granu) Polygon if the side is known. 6.3.1: Area of a polygon having side a and a sides =—"® 180) Atan(— n va F the area of an equilatera triangle having side a is 77 = “52 4tan6o 4 6.4 REGULAR POLYHEDRA & EULER’S FORMULA A solid figure whose faces are polygons Is called a polyhedron. Examples are a cube, a tetrahedron (all four faces are triangles) and the two polyhedra below. h\ xD Fig. 1 Fig.2 Every polyhedron has three numbers associated wit it: E=number of edges F = number of faces \V= number of vertices. Fora cube, (F, V, E) = (6,8, 12} For a tetrahedron, (F, V, E) = (4, 4, 6) In Fig 1., (F, V, E) =(7, 7,12) In Fig. 2, (F, V, E) = (5, 6, 9)- Interestingly, there is a connection between F, V and E for Leonard Euler. all polyhedra. This was first observed by 6.4.1 Euler’s Formula: F+V=E+2 Check the truth of this formula in the four examples above. Remember it! 6.5 PLATONIC SOLIDS You are quite familiar with a cube; a die, with 1, 2, 3.4, 5, or 6 dots on each face has the shape of a cube. All the faces are identical squares. 6.5.1 Definition: A Platonic Solid is a solid object all of whose faces are congruent polygons and with the same number of faces meeting at each vertex. ‘A cube is an example of Platonic solid. Another example is a tetrahedron. Allits (four) faces are equilateral triangles. Impossible as it may seem, we can actually prove that there are exactly five Platonic solids, two of which are the cube and the tetrahedron above. 6.6 THERE ARE EXACTLY FIVE PLATONIC SOLIDS Proof: Let P be a Platonic solid. Let F=ti(number of) faces, \V=# vertices, Es edges, p= # edges surrounding each face, = Ht edges radiating from each vertex. (For example, ifP is the cube, then Fe6, V=8, E=12, ba Obviously p23 & G23 (y) From Euler's Formula: FAV = £42" Count the total number of edges, using the vertices: q edges radiate from each of the V vertices. Altogether there are qV’ such edges, but each edge is counted twice. It follows that qV=2e .. (3) Count the total number of edges, using the faces: there are F faces and each face has p edges. Altogether there are pF such edges, but each edge is counted twice. It follows that: pF =2E 4) Substitute (3) and (4) in (2): 26 4 2£ pa +2, Hence 2q+2Ep = Epq +2pq E(2p+2q—pq)=2pq 24 2p+2q~p4 204 eel) 4-(p~2q~2) i Now (p- 2)(q- 2) is a natural number it cannot bezero else (1) would be contradicted - and so is E Hence (-2\q-2)€(1,2,3) Also, each of p—2 and q~2 is a natural number. The table below lists all the possibilities (we make repeated use of equations (3), (4) and (5) in populating the table, @-2@2)]-2[4-2| pa] 2a] 4—@-2a-2) [E | yu2e | Tre 1 1 1 3/3/18 13 6 4 4 Tetrahedron 2 1 [2 |3|/4]m [2 @ [8 6 Ocahedron 2 2 | [4|/3}a [2 @ [6 3 Gabe 3 a fs [3]s}ao fa 30 [0 a2 icosahedron 3 3 1 5/3]30 /1 30 | 12 20 Dodecahedron: Extracting the relevant information from the above ‘able, we get: plale |F |V | Type 3/3[6 |4 |4 | Tetrahedron 3/4]12[8 [6 | Octahedron 4|3fa2[6 [a | cube 3/5 | 30 | 20/| 12 | tcosahedron 5/3 | 30 | 12 | 20 | Dodecahedron dodecshadvon seotahedron 6.7 APPLICATIONS OF LESSON 6 104 [205 277 37 LESSON 7: PYTHAGOREAN TRIPLES Recall (see 3.7), that two integers have the same parity if either both are even, or both are odd. The sum (and difference) of numbers having the same parity is always even, 7.1 TWO USEFUL IDENTITIES It can easily be checked that the following identitieshold true for all numbers a and b: a+b) 2 7.2 INTEGERS THAT ARE DIFFERENCES OF TWO SQUARES An integer is a difference of two squares if, and only if, it is a product of two numbers having the same parity. (2) Proof: if a and 6 have the same parity, the bracketed numbers in (2) are integers, so one direction is clear. Conversely, let x=a" ~b” where a and b are intege’s. Then xis the product of the two integers a= band a +b, whose sum (namely, 2a) is even, so they have the same parity. For example, 120 = 1.120 = 2.60 = 3.40 = 4.30 = 5.24= 6,20 = 8.15 = 10.12 gives us all the factorisations of 120. Of these, four pairs of factors, namely, the second, fourth, sixth and eighth, have the same parity. Applying (1), we obtain 60+2 120 +29" = 31”. =29 & Stan 1204 13%=17* 12047? = 13" 120+ 17 = 11? 7.3 CONSTRUCTING PYTHAGOREAN TRIPLES, GIVEN A SIDE Every square number N? > 4 has a factorization inte two numbers having the same parity. This is easy to see; if N is odd, N? = 1.N? is a product of odd numbers, and if Nis even, N? (<2 gives such a factorization, which lead to the equations: weet) (if N is odd) if N is even) ‘A Pythagorean triple is a triple (x, y, 2) of integers y and z, where 2 is the hypotenuse of a right angled triangle having sides x and y; alternatively, if x, y and z are integers such that x” +-y" ‘The above discussion allows us to come to the following conclusion: 7.3.1 Every integer N > 3 is a side (not the hypotenuse) of a right angled triangle with integral sides. =b arb ‘The Pythagorean triples are precisely (1 > a) where N?=ab isa factorisation of NV? into two integers having the same parity. 7.3.2 To find all right angles triangles having a side a (different from the hypotenuse), proceed as follows: 1. Calculate a. 2. Write a” asa product a” ¢y of two numbers having the same parity. 2 (XV _(xtvY (0 xey ot 3. Then a? +{*—¥) =(X*Y) oo [ g,%-¥ **Y )ic a pythagorean triple. 2 2 272 For given 0, to find all integers m and n such that (m,n, ) is @ Pythagorean triple is @ much more complex problem, and we shall not deal with this question. We will remark, however, that there need not be a solution for m and n, for certain values of a. 7.4 CONSTRUCTING PYTHAGOREAN TRIPLES, GIVEN THE HYPOTENUSE Unlike the “side not hypotenuse” case, there are infnitely many integers that are not the hypotenuse of a rightangled triangle. The following is stated without proof. 7.4.1 If among the prime factors of a, there is none which, when divided by 4, leaves a remainder of 1, or put in another way, if a has no prime factors ofthe form 4n + 1, then there is no Pythagorean having a as the hypotenuse. Otherwise there are ahvays such triples. Hence, numbers like 7, 21= 3.7, 209= 11.19 cannct be an hypotenuse of aright angled triangle. But 55 = 5.11 and 247 =13.19 can be. Check that 33, 44, 55) and (95, 228, 247) are both Pythagorean triples. 7.4.2 Examples: 1, The integers 5, 13, 17, 29, 89, 1109 are all 4n + 1 primes. The first five are easily seen to be sums of of, two integral squares. Note that 1109 = 22° + 252, 2, 387 = 37.43 cannot be written as a sum of two sqlares since its prime factor 43 is of type 4n +1 and has, ‘odd exponent ( =1). 3.55 is an hypotenuse of a right angled triangle wity integral sides: 55? = 11°(25) = 11°(3" + 42) = 33744? Note that 55 has factor 5, which is a 4n + 1 prime However 209 cannot be the hypotenuse of a right angled triangle with integral sides since 209 = 11.19and both 11 and 19 are 4n +3 primes. 39 4. Find all right-angled triangles having 36 as one of ts lateral sides. Solution: 36° =2.2.2.2.3.3.3.3 has the following adnissible factorisations, 2.648 4.324 6.216 8.162 12.108 18.72 2454 ‘There are seven triangles having 36 as one their smaller sides. From (2), their sides are, respectively, (36, 323, 325), (36, 160, 164), (36, 105, 111), (36, 77 85), (36, 48, 60), (36, 27, 45), (36, 15, 39). 4. The following identities are worth remembering: (a +b’)(c’ +”) =(ad -bc)’ +(ac+bd)? (a + bc? +d?) =(ac—bd}’ +(ad + be) We deduce from them that a product of ums of two squares is again a sum of two squares, possible in two different ways. 5. 530 = 2.5.53 has no 4n + 3 prime in its prime factorisation. 530=2(2" +1')(7" +2") =22.7+1.2F +(2.2-1.7)] =216 +3") =(+1)06" +3") = (16-3) +(16+3)° 13° +19" Similarly 530=23? +1? 7.5 APPLICATIONS OF LESSON 7 cS 66 286 LESSON 8: USEFUL FORMULAE 8.1 SEQUENCES 8.1.1 Definition: A string of numbers separated by commas is called a sequence. 8.1.2 Examples: (a) Allnatural numbers: 1, 2, 3, {b) _Allssquares from 5 onwards: 25, 36, 19, 64... (c)__Allodd cubes from—3 upwards: - 21, -1, 1, 27, 125,, ‘The numbers in a sequence are called terms. Thus the third term in (b) above is 49, for example. (d) How many terms are there in the sequence 34, 35, 36, .., 237? Anneat way of answering the question is to include tie numbers 1 to 33 in the sequence and then remove them! So the answer is 237— 33 = 204. In general, 8.1.3 There aren -(m—1) numbers between mand n,, both inclusive. Thatiis, the sequence m, m+ 1, m+2,..., n has n—m+ 1 terms. We can easily see why. If we include the m -Lnumbers 1, 2, 3, .., (m-1) in this sequence, then we have n terms. Now remove them, to deduce that the original sequence has n - (m— 1) —m+ 1 terns. 8.2 USEFUL SERIES 8.2.1 Sum of the first n natural numbers 142434..n- 2040) See Lesson 5.1.1, problem 1, for an explanation. This is the formula for the sum of the first n natural numbers. Itis also the n” triangular number. 8.2.2 Sum of the first odd numbers ‘The sum of the first n odd numbers is n?. Proof: Let us check whether the claim Is true for thefirst few values of n. a1 143 143452923 143+5+7=16=4%, ‘This test , of course, does not prove the statement. t only verifies it for the first four values of n. How can. we be certain that if we add the first million odd numbers, for example, we will get the square of a million, that is, 1.000 000°? 1 So, we have to verify the claim for every natural nunber n. Method 1: Let x be the sum. That is, x=14+3+5+- ton terms. Then x= (2-1) (4-1) + (6-1) += (m brackets) = (24446 4..)~(141 + 16...) (where each braccet has n terms) =2(142+34 224274) _ from (2) above n+n-n . Method 2 In the sum x=1+3+5+-, insert the first n even numbers and then subtract them: x=(1 +(2) +3 #(4) +54 (6) + (24446 +) The first bracket is the sum of the first 2n natural numbers and, by problem 2, itis equal to 22/2") The second bracket is, as before, 2 times the sum ofthe first n natural numbers, so itis equal to 2 nina) Substituting: 2n(2n+1) 2 nna nln+-1) Method 3: Note first that the terms of 1+3+5+-~ are 2(1)- 1,2(2)~1, 2(3)~1, .. So the n® term is, 2n-1. eLIBHS 474 #21) nn () Now write the same sum backwards. x= (20-1) + (2n=3) 4454341 ‘Adding (1) and (2): xx= 2n+2n4 2n += (nterms) = n(2n). Hence 2x = 2n? sox=14+3+547 4+ +(2n-! (2) 8.3 ARITHMETIC SEQUENCES The three sequences, namely, the natural numbers,the odd numbers and the even numbers, all have a ‘common property. Each term in the sequence from the second one onwards, can be obtained from the previous term by adding a fixed number. 92 Another way of saying this is the difference between consecutive terms (taken inthe same “direction”? is always the same. ‘This is difference is called the common difference. Sich a sequence is called an arithmetic sequence. 8.3.1 The n® term of an arithmetic sequence Let a be the first term of a sequence which has theproperty that every term is obtained from the previous one by adding a fixed number d. The n"* term of an arithmetic sequence with firstterm a and common difference dis a+(n—1)d. Proof: The first term is a, the second term is a +d (adding d to a), the third term is a + 2d (adding d to +d), the fourth term is a + 3d (adding d to a + 2d). "roceeding in this way, the n" term is a+(n—1}d. 8.3.2 The sum of the first n terms of such an arithmetic sequence Ia is the first term of arithmetic sequence which hes common difference d, the sum of the first n terms is given by: a+(0-+d)+(a+2d)+.....to.n terms Proof: Let x=a+(a+d)+(a-+2d)+. Then x=(a+a+at--+a)+(0+142+3+...)d where each bracket contains n terms. Which meansthat the second bracket is the sum of the first n-1 natural numbers. X=(atata+-+a)+(0+1424+3+...)a =na+[(1+2+34---+(n-1))d (n-1)0 =na+ d fromS.1.1 =F 20+(n—1). Hence a+(a+d)+(a+2d)+....to n terms = 8.3.3 Example: (a) the 300" term of the arithmetic sequence — (b) the vatue of -% 93 Solution: (0) The fst term ofthe arithmetic sequence is a=~5 and the value of dis ~13+5°=2. (We can check that each of the given terms is obtained by adding 3 of the previous term). The 300" term (son = 300) is a+(n=1)d = So, the above formula may be used, with required sum is 200 9 F-135+(200 -1(2} 8.4 SUMS OF SQUARES OF FIRST n NATURAL NUMBERS We have seen that there is a quick way of calculating the sum of the first n natural numbers. Simply use the formula 14243+--4n= M049) - What aboutthe squares of natural numbers? Or cubes? Are there formulae for the sum of the squares, or, for that matter the sums of the cubes of the first n natural numbers? Let's start with the squares and see whether there it a pattern: P42 =144=5 Pa 43 =549=14 242? 43°44? =14+16=30 P4243 44? +5? =30425=55 At first glance, there does not appear to be any wayof predicting what the next sum is going to be: 1,5, 14, 30,55, 7? If we divide these numbers by the sum of the assocated numbers, we get 15 14 30 55 13’ 6'10'15 Which when simplified become Poa 3°3 ‘Summarising, it would appear that; VPs eutnh ned 14253+..40 2 and this is just the same as 9 3 , and here there Is a clear pattern! forn=123,45. nin+1) ‘The denominator on the let is, as we have seen, equal to so we suspect that: ats cae (2082) nln 2ne3) | 2 3 6 We cannot be certain. We know only that it works for n = 1, 2, 3, 4 and 5. So how we can verify that that the formula holds for every sum of n squares? There is @ way. Avery clever way. We start by making the following observation: Let k be any number. (e447 0 437 43k 41-0 3 23k (41) BEBE conn) Now take any natural number n. Make n substitutions for k in the equation (1), namay k | k= 2, k= 3, ...k =, but do not simplify. P-VaBL +3141 3-2 93.243.241 4-3) =3.3° 43.341 Soa) 23.4 43.41, (n+ 1) -n? =3n? +3042 Now add these n equations. All but two terms disagpear on the left hand side. On the right, remove 3 as ‘a. common factor from the first two columns and note that the sum of the n ones isn. (41) 1-30? +243 44? bot L4 2434-4 Men nn+1) 2 2p? +3n? +30) = (1? +2! 43° 448 4-4 m4 3nla+2) +20 6(2? 42? 43? 4.4? +--+?) =2nln? +3n+3)—3n(n+1)—2n 60? 42°43? 44? 4-4?) nla? +604 6—3n-3— GP +243? 44 4-40") =nln? +3041) 60 4243 44? fon?) =nln41)(2n+1) _nla+1)(2n+1) =neenened) 4 3n? + 3n4 11H 4243 44? +--+ 3: +n (42°43? 44? 40+ +n?) We have proved: > _aln+1)(2n-2) 6 VHP sea geen’ 95 8.5 SUMS OF CUBES OF FIRST n NATURAL NUMBERS The sum of the cubes is even more interesting, and surprising, Let's see whether there is a pattern. VP 42) =148=9 142? 43? =9427=36 P4243? 44? =36+64=100 142° 43° +4? +5? =1004125=225 Here the sums show a clear pattern: 1°,3”,67,10°,15* which is just 042) (142437 (1424344) (142434445) ‘Summarising, we have the fascinating formula 85.1 P42 4s sa peg (1424344 bet nl? i ‘As before, we cannot be certain that the formula werks for all possible values on n. The proof is identical to the one for squares: start by simplifying (k+4)*—K', 8.6 GEOMETRIC SEQUENCES Consider the sequences (a) 1,2, 4,8, 16, (b) 10, 100, 1000, 10000, ... @ 2,-2,4, -4... 5 5 15 45 They all have the same property: each term from the second term onwards can be obtained from the previous term by multiplying it by a fixed number. ~he fixed number in (a) is 2, in (b), itis 10, and in (e), it Alternately, the ratio of consecutive terms (taken inthe same direction), is the same throughout the sequence. Its called the common ratio of the sequence. ‘Such a sequence is called a geometric sequence. ‘As before we let a be the first term. Let the common ratio be r. The geometric sequence is therefore: 9, ar, or’, ar’, It should be clear that the n™ term is ar”. Proof; (a) Let x be equal to the sum. The last term on the rightis 1”"'. So KaLE EP EP te Multiply both sides by r and subtract: KEL EP AP ete me Pt ete (b) Simply multiply both sides of the above equation by a. 8.6.2 Factorisation of 1-1” (EP EP et Follows immediately from 8.5.1(a). 8.6.3 Factorisation of a" ~b" For any natural number n, (a—b)(a""* +a"*b +a" Pps B) To prove this, let =— in 8.6.2 and multiply both sides by a” . Note that a"(1-r")=a(1-r)a" tr tr? 4-41"). Youcan fill in the details. 8.6.4 Factorisation of a" +b" for an odd number n. a" +b" =(a+b\(a"*-a"*b+a"*b + +b") To prove this, note that for n odd, a” +b" = (-5)" so we can use 8.8.5, by replacing b in that ‘equation with ~ b. Note that, since n-1 is even, it folows that (-b)"" =b"" Important Fact: It follows from 8.6.3 and 8.6.4 thatfor integers a and b: 97 ‘a~ bisa factor of a —b* for all natural numbers n a+ bisa factor of a+ b" for odd natural numbers r. 8.6.5 Examples: 1. a? —b? =(a-byfa+b) —b* =(0-bya? +-ab-+b") a —b* =(a-b)la? +0°b + 0b? +b*)=(a—b)(a+ bo" +6") 2, a? +b? =(0+6)(a" —0b +b") a’ +b =(o+b)(a" -a'b+<°b' ~ab" +b*) 3. Determine at least one non-trivial factor of 3°" +8". Solutioy 3 +g! =(3°)!" +2"), Since 1007is odd, we have, from 8.8.6 that 9 + 8 = 17 isa factor of this number. 8.7 OTHER USEFUL FORMULAE: The following are stated without proof — they are easy to check ~ and should be committed to memory. B71 x ty +2 —Bayz—(xtyt2)oe ty +2 —xy—yz—12) 87.2 (xtyt2) =x ty 42? +2ay tye +2en 2 2 Liqyy yz? 8.73 (Psy tz’ Veen = lx —¥F +y-zP +(z-xF] 8.7.4 a”—b’ =(0-b)(a"* +a"*b+....ab™ +b™) 1 875 (0b) =c,0" +¢0"b+e0"%"+...+¢,8° where ¢, =(""]=—"_, 1 @-nirl 8.7.6 (a+b) =a" +30°b + 3ab* +b* 8.7.7 (a+b)' =a +4a°b+6a'b* +4ab* +b* 8.7.8 x+y" +2 —6xye=(x4y+2)(x" +y? +2! -xy—ye—20), 8.8 The AG (Arithmetic-geometric inequality) 8.8.1 | The arithmetic mean, that is, the average, of two non-negative numbers a and b can never be less than their geometric mean. That is, for any two numbers a and b, his ae 2 lab . Moreover, they are equal if aud only ifa=b. Thats, the average of two positive numbers (also called the arithmetic mean of a and b) is at least as large the geometric mean of the two numbers. Thisinequality is called the AG inequality. Note that © = Jab i and only if (obj =0, thas, a= b In general, we have: For any n positive numbers o,,0,, 0,,.. Get, + at +0, (ettet tes aoa, ” with equality if and only if a, =a, =a, =. 8.9 APPLICATIONS OF LESSON 8 1 14 17 2 28 a 80 92 95 126 155 156 181 199 218 243, 253, LESSON 9: PERFECT NUMBERS Let m be a natural number. How many (positive) factors does m have? The answer is given in Lesson 1.5. There itis shown that if n=p,"p,"p,".... is the factorisation of n into prime powers, then n has (a+1) (b + 1)(c +1) ..factors altogether. 9.1 THE SUM OF FACTORS OF A NUMBER We now turn our attention to the sum of all the factors of a number. For example the sum of all the factors of 15 is 1+ 3 +5 +15 = 24. The symbol cr (n)is used for the sum ofall the factors of n. (Note that c is a function having domain the set of natural numbers). Thus (15) = sum of the factors of 15 = 14345415 = 24. Let us calculate or (400), 400 =2'S? has 5.3 = 15 factors and they are: 4 2 2 2 2, 5, sat, 52, 52°, s2t, 5, St2', St2*, Sta, *st2', @ (00)= (1+ 2+2°4 284 24) + 5(1 42+ 2242942) +541 +2 +2424 24),...cuiremove common factor (1424224294 245 454) This isa product of two geometric series. So from 8.5.1, 2-18-21 =o(2*)ot5" (2B) ‘The idea in the above example allows us to calculate the sum of the factors of any number. ols 9.1. ofp'a'r The function cr has the following useful property, ais illustrated by a careful examination of 9.1.1: 9.1.2 Ifa and b are coprime (see Lesson 2.4), then o(ab)=cla)oib) 100 9.2 PERFECT NUMBERS. 9.2.1 Definition: A number is said to be perfect if itis equal to the sum of all its proper factors. (A proper factor of a number is one that is different fram itself, so 1 is a proper factor of 6 for example, but 6 is not.) Let m be perfect. Then the sum of its proper factors & equal to n. Hence the sum of all its factors = n += 2n. That is, ifm is perfect, then or (n) = 2n. Hence: 9.2.2 Anumber is perfect if o-(n) =2n The first three perfect numbers are 6, 28 and 496. Itis easy to check that they are perfect: o(6)=1+24+3+6=12=26 @(28)=1424+447414+28=56= 2.28 (496) = o(2*.31) -( 252 )u3n-s92-a090 (a If we examine these three numbers more closely, we see that:- 6 +23 =2(2-1) 28 247 =2%(2%-1) 496 =16.31 = 2'(2°—1) So there appears to be a pattern, namely that 2° (:"*! ~ 1) is perfect. However 2(2*~ 1) = 120is not perfect, as can easily be checked. The reason for this is that 21 = is not a prime number. 9.3 A CLASS OF PERFECT NUMBERS 9.3.1 The number N=2"(2""—1) is perfect whenever 2""' — ‘a prime number. cn Proof: Let p = 2”*—1 be a prime number. Then ~1)=2" p. Now 2° is even and pis odd, so these factors of N are coprime. From 9.2, a1) =a'2"p) =ol'boto) = ( = Jas p) = (2? -1)(1+2""-1) =2.2°@"-1) =2N Thatiis, Nis perfect. For n=1,2,3,4,5,6,7,8,9, 10, 11, 12, 101 20-1 = 3, 7, 15,31, 63, 127, 255, 511, 1023, 2047, 4095, 8191 and of these only 3, 7, 31, 127 and 8191 are prime numbers. And these yield 5 perfect numbers, In this way we can identify a whole group of perfect numbers. Of course, there may be others. 9.4 CLASSIFICATION OF EVEN PERFECTNUMBERS, Indeed, it turns out that the above collection of perfect numbers exhaust all the even perfect numbers. 9.4.1 If nis an even perfect number, then n=2'(2*—1)for some r21, where 2'*—: number . aprime Proof: Now let n be an even perfect number. Then o(n)=2n Write nin the form n=2q where r21 and qis odd. (see 1.3.4)... Then, since nis perfect, o(n)=2n=2""q, ~ (2) But n (0) = (2 Jo(Q)= (2° -1}0(g) «sete eeninnnreernnee() From (2) and (3), it follows that (2 —aola)= 2"*q intonation A) Now 2""'—1is an odd number that is a factor of 29 . Since 2"*—1and 2" are consecutive numbers, it (1) " q isa product of coprime numbers so fram 9.2, follows that 2°"*—1 is coprime to 2”. By 2.4.2, 2” q=(2"""—1}¢for some integer t> 0... 1 Is a factor of g. So (5) Assume t > 1. Substitute (5) in (4) to conclude that o{q) =2"*tt From (5), 30 olq)=t+q =2"t-t=olq)—t (6) From (5), tis a factor of q. Further t > 1 by assumption. Can t = q be possible? If so, from (5), 2" =1, sor =-1,a contradiction. Sotteq Contradicting (6). Our assumption > 1is thereforefalse, that is, = 1. Hence from (5) Land from (4) (7) So q has exactly two factors, namely 1 and q, that is, q= 2/*" —1 isa prime. Finally from (2) n=%q=2(2"—1). which is what we set out to prove. 102 ‘The even perfect numbers are therefore completelyknawn. To date, no odd perfect numbers have been found, and there is a belief that there are no odd perfect numbers. No one has been able to prove this, however. If you are able to, you will become world-lamous overnight, as this is a famous unsolved problem, 9.5 APPLICATIONS OF LESSON 9 103 LESSON 10: THE FERMAT POINT The Fermat point of a triangle, also called the Torricelli point, is a point inside the triangle that has the property that the sum of the three distances from the point to the vertices is as small as possible. Itis so named because this problem was first raised by Pierre de Fermat (1601-1665) in a private letter to Evangelista Torricelli, who solved it. 10.1 THE PROBLEM: Find the point P inside | ABC so that the sum PA + PB + PCis as small as possible. a We first remark that if one of the angles of the triangle is greater than 120’, then the point P we seek lies outside the triangle, as we shall see later. So there is no point inside the triangle that makes the sum as small as possible. Let us assume that all the angles of the triangle are less than 120°. ‘Method 1: First, take any point Q inside the triangle. We need to find the smallest possible value for QA + QB +QC. This method tries to link the three lengths into a “jagged” line, so that for minimum sum, the jagged line has to be straight. This allows us to lacate P quickly. The trick is to rotate _| BQA through an angle of 60 around 8, in an anticlockwise direction as in figure, to produce the_| BQ’A’, (Q and A go to Q! and A’ resp. via the rotation). These triangles are therefore congruent. Further BQ= BQ’ and 4 QBQ'= 2 ABA’= 60°. So_| BQQ’ is equilateral. Also QA = Q’A’, by congruence. So QA +Q8 + QC = A’Q + Q/Q + QCis ust the length of the “jagged” line A'QQYC, and its length is smallest when A’QQ’C isa straight line! 104 ‘That is, Qis a point on the straight line CA’, for mininum sum. How is A’ obtained? Notice that since BA= BA’ and ZA’BA= 60) ,_| ABA’ is equilateral. £0 to get A’, we construct the equilateral triangle ‘ABA’ on AB, where A’ and C are on opposite sides cf AB. ‘Summarising: The point Q that gives the minimum sum QA +QB +QC. ‘equilateral triangle, with F and C on opposite sidesof AB. son the line CF, where ABF is an F B ° Now, clearly, there is nothing special about the side AB of_| ABC. We could have constructed the ‘equilateral | CAE on AC, with B and £ on opposite sides of AC, joined BE, and declared that Qs also on BE. B c Hence, the point O is the meeting point of BE and CD. ‘Another interesting fact is that Z BOC = 120°. Proof : First notice that | AEB=_| ACD . This is because: AE=AC(_! AECis equilateral), ‘AB = AD (_) ADB is equilateral) and ZBAE=60° + ZBAC=DAC. 105 so, |AEB=lACcD (s, 2,3). Hence ZABE= Z ADC. But both these angles are subtended by AO. Which means that BOAD is a cyclic quadrilateral, Hence 8D subtends equal angles at O and A. Since 2 BAD = 60, we conclude that “BOD = 60°. But DOC is a straight line, so 2 BOC = 120". But again, there is nothing special about BC. The same ought to be said about AB and AC. That is, ZAOB= ZAOC= 120°. ‘Summarising: The point 0 is at point where the three sides of the triangle each subtend 120°. Method 2 Firstly, we note an interesting property of equilatera| triangles. 10.2 NAPOLEON'S THEOREM Let X’Z be an equilateral triangle. IfP isa random point in the irterior of the triangle, then the sum of the distances, fromP to the sides of the triangle is always the same, regardless of the choice of P. Moreover this constant sum is equal to the height of the equilateral triangle. Pro Refer to the figure above. We need to prove that 2X. The area of triangle XYZ is the sum of the areas of triangles PXY, PYZ and PZX. That is, Jah, +4ah,. Itimmediately follows that h, +h, +h, 106 Next we give an alternative proof that the point Q mentioned above - where all the angles are equal to 120°, has the property that QA + QB + QC is less then or equal to PA + PB + PC for any other point P inside the triangle. Let C be the point inside the triangle at which Z AQB = , Zpac= ZcaA=120". At Adraw XAY 1 QA. Likewise, YCZ and ZBX are ZN perpendicular to QC and QB respectively. In quadrilateral e ve anv, Z¥+90" +90° +120" =360'. Hence “Y= 60 Simibrly ZX=22 =60 ‘Thats, triangle XZis equilateral Now let P be any other point in the interior of triande ABC. We need to show that PA+PB +PC 2 QA+QB+0C Drop perzendiculars PR, PS and PT to the sides of the triangle, {as showr. As hypotenuses of right angled triangles, PA, PB and. PC are ali greater than PR, PS and PT respectively. It follows that PA- PB+PC> PR +PS + PT. @ However, by Napoleon's Theorem and the fact that XYZ is equilateral, we have that PR+PS+PT=QA+QB+QC (2) Combining (1) and (2), PA+PB4PC> QA+QB+QC, and this is what we set out to prove. 10.3 APPLICATIONS OF LESSON 10 223 | 236 107 LESSON 11: TRIANGLE FORMULAE 11.1 THE SEMIPERIMETER OF A TRIANGLE In what follows, A ABC will have sides a, b and c, and these will be opposite angles A,B and C respectively. By the triangle inequality, a+b>c, b+c>aande+a>b. (2) e 2 c Soall of a+b—c, b+e-a & c+a—bare positive real numbers. a+bte The perimeter of A ABCis a +b +c and its semiperimeter is s = This quantity plays a very important role in calculations, as we shall presently see. It is easily seen that ~atb+e 2 a-bte (2) and therefore, another way of writing the triangle inequalities is, 5-020, $—b>0, $—b> Dvn renner) In this lesson, we shall calculate three important corstants associated with a triangle, namely, its area, the radius of its inscribed circle (called the incircle) end the radius of the circle that passes through the three vertices, that is, the circumeircle of ABC. Allthese will be calculated in terms of a,b and c. The symbol Ris used to denote the radius of the circumrcle of A ABC. 11.2 THE SINE RULE The sine rule for a tiangle says that the ratio of a side of a triangle to the sine of the angle opposite it is the same, regardless of the angle selected. More precisely, 2 c 108 a bic sinA sinB sinc ‘Actually it says a little more, namely, that this constant number is the diameter of the circumscribed circle of A ABC. Refer to the figure alongside. The circumcentre A ABC is O, and the construction is to draw the diameter AD through A. So AD = 2R, Now, a diameter of a circle subtends a right angle at the circumference, so angle ACD = 90°. So A ACD is a right angled triangle andsinD -. so 2R. But angles B and D are both subtended by the same chord, namely AC. Therefore b 2B= LD ane | = 2R = diameter of crcumeircle. In the same sin way, the other two ratios can also be shown to be equal to the diameter. So ‘THE SINE RULE: ~~ sina (3) sinB sin 11.3 THE COSINE RULE There is also a cosine rule. itis a rule whereby the cosine of any angle of a triangle can be expressed in 8 ax Dee terms of the three sides. The constructior here is to draw the altitude AD. Let x= DC. Then BD = a—x. Using the theorem of Pythagoras, we have the following: AD? =C ~(a—x) =b* -x? For each side of the triangle, there is a cosine rule: of =b' +c? ~2becosA B =a" +b? ~2abcosC 2 +a" —2eacosB (4) 109 The sine and cosine rules hold also for obtuse anglec triangles, although our proofs were only for acute ‘angled triangles. 11.4 AREA FORMULA (1) In what follows, the symbol A, which usually reads “triangle”, will also mean the area of a triangle. We are familiar with the formula that gives the area A of a triangle as one half the product of its base and height. That is, A=4ah, where a and h are as in the igure above. (5) Now 7 1csin B . This gives the area of the triangle, given two c sides and an included angle. There is nothing special about angle 8; there are three such formulae: . in B, so h=csin B. Substitute to get A 11.5 AREA OF A TRIANGLE (2) : HERON'S FORMULA What is the area of a triangle whose three sides are given? Heron's formula answers this question, HERON'S FORMULA: The area A of a triargle whose sides are a, b and c is given by: A=\sis—a)(s—B\s—0.. a+bte, | where Proof: From equation 5, A=absinc so Pb? sin? C ?b'(1-cos*C) =4a'b'(1-coscl(1+cosc) a ee pe (from equation 4) 2ab 2ab =P (eob—a? -b? +c')\2ab+0" + bc") 16a°b’ =4[c?-(o*2ab+b*)|[ (a +2ab+b')—c'] =4[e--oF)][lo+b -<] (c+a-b\e-a+b\a+b+cl'a+b-c) a+b+c)\(-at+b+c)\(a—b+c\(a+b-c CREE) =s(s—o)(s—b)ls~¢). rom equation (2) Hence A=,/s(s—a)(s—b)(s—c) no 11.6 RADII OF THE CIRCUMSCRIBED AND INSCRIBED CIRCLES 11.6.1 The radius of the circumscribed circle Recall that R= -from equation (3) Hence from equation (5) abe (7) 2ysls-ay(s-bys-¢) 11.6.2 The radius of the inscribed circle The three bisectors of the angles of a triangle meet at one point, called the incentre . It is usually denoted by. If, from |, perpendiculars are drawn to the three sides, they all have the same length, say, r. The circle centred at | and having radius r, touches all three sides — the sides are tangent to this, Circle. It is this r that we shall now calculate. IF A denotesthe area of the triangle, we see that Ais the sum of the areas of three triangles, which are swept out when I's joined to the three vertices. Ifa, b and care taken to be the bases of these triangles, then they all have the same height, namely, r. We can conclude dor+ibr+der: (3 \rawr Hence Assr A_Vs(s=a\(s—b)(s—c) 5 s using Heron’s formula. 11.7 THE TRIANGLE INEQUALITIES Finally, the quantities s~ a, s~b, and s~ c turn outto be precisely the distance between the vertices of the triangle and the points of contact with the inscribed circle. More precisely: Let x, y and z be the distances from the vertices to the points of contact. (See diagram below]. We note that the two tangents drawn from a point outside the circle have the same lengths. mi x From the diagram, the perimeter of the triangle is 2x + 2y + 22, But the perimeter is also twice the semi-perimeter, that is 2s. Hence 2s=2¢+2y+22. xtyte=s, x=s-a y=s-bb. zas-c ‘The triangle inequality says that the sum of any twe sides of a triangle is greater then the third. Thus if a, band care the sides of a triangle, we always have: b+e>a, cru>b, arb>e But The triangle inequalities are therefore the same as: x>0, y>0, 2>0. 11.8 APPLICATIONS OF LESSON 11. 3 368 [200 [200 [227 [236 [261 [265 [277 [285 LESSON 12 (TRIANGLES 2, RATIOS & AREAS) 12.1 WHY IS THE AREA OF A TRIANGLE EQUAL TO “HALF BASE TIMES HEIGHT”? It is well known that the area of a triangle is obtained by halving the base and multiplying it by its height. We first trace the roots of this formula. m2 In what follows the notation (ABC...) will mean thearea of the region ABC... Let ABC be a triangle having height AD = h. 1 We shall prove the area (ABC) BCh 2 To this end, complete the parallelogram BCEA and ectangle BCFG, where F,G and H are ona line though A parallel to BC. 3 ar E | > ‘The shaded triangles are congruent (s, 2, s) and so 1ave the same area. To each of these areas, add the area of the trapezium AFCB. We see that (BCEA) = (BCFG) Now, the diagonal AC divides the parallelogram BCES into two congruent triangles — one of which is triangle ABC ~so each of these triangles has one half the area of the parallelogram. Also BCFG is a rectangle, by construction, so its areais length (BC) times breadth (height h). Summarising: 2/ABC) = BC.h proving what we set out to prove, nanely, (asc) ; BCh. It follows that if two triangles have equal bases and reights, they will have the same area. 12.2 TRIANGLES BETWEEN SAME PARALLELS AND HAVING SAME BASE HAVE EQUAL AREAS 12.2.1 If line through a point A is parallel to BC, then for any point P on the parallel line, (PBC) = (ABC). m3 12.3 A MEDIAN BISECTS THE AREA OF A TRIANGLE Firstly, we make the following important observation: if Sis P any pont on side RQ of triangle PAR, then 5 — (RFS) (SPQ) Proof: Vote that triangles RPS and $PQ have the same height fh. Then BS _ 285 _ (RPS) | SQ 3SQh (SPQ) ‘An immediate consequence is: A median bisects th: area of a triangle. x Fromihe above, a tose" c8~ Ac (DAZ)=(DBC) That is median DC bisects the area of triangle ABD. 12.4 AN ANGLE BISECTOR RATIO ‘Theorem: If PS is the bisector of ZP of APOR, then Proof; £5. (R57) SQ” (SQP) JPRPSSINC from 11.4 3PQ.PSsinx PR 12.5 The Mean Proportion Theorem 12.5.1 Auseful fact about ratios a_x Let “=~ and let p and q be any two numbers. Let { Then a=kb & x= 6 na pa+gx _ pkb+aky _, _@ pbh+qy — pb+ay by Summarising if “=~ and p and q are any two real numbers, then 7 =~ 7). y by In particular, for p |, and =p = Land q=-1, wehave a@_x_a+x_a-x boy bey by We also have att? xy 12.5.2 The Mean Proportion Theorem Aline parallel to a side of a triangle divides the theother two sides in the same proportion. a The theorem says that if XY [|BC, then <= 4¥ xB YC The result is an immediate consequence of equatiors (2) and (3): AK _(AXY) xB (xBY) v6) from (3) from (2) soeuftom (3) The same result holds for any point X on the extended line AB. More precisely: ns XA Even in these cases the equation “~ =" is true. xB YC 12.5.3 triangles are similar if and only if their corresponding sides are in the same proportion, AB _ BC _ cA DF DE EF Notes: To verify that two triangles are similar, we nzed only show that two angles of one triangle are equal to two angles of the other. The remaining anges will then be automatically equal. 1. Inthe figure, the sides opposite angle x, namely CA and EF, are corresponding sides. Likewise, AB and DF are corresponding sides since both are opposite angle y. 12.6 CEVIANS 12.6.1 Auseful ratio Let PS be a Cevian of triangle POR. (A Cevian is a lin2 segment joining a vertex of a triangle to any point on the apposite side.) We saw in 12.3 that (asp) _3RSh_ RS (saP] 3SQh SQ ne ‘This equation has a number of applications. We discuss some of them, In the diagrams below, O is any point that is not on any of the three sides of AABC. It need not be inside the triangle. 5 ; / a E 9 8 D c a . © Fig. Fig. 2 The point O determines the Cevians AOD, BOE and COF (or CFO in figure 2). 12.6.2 Ceva’s Theorem AF BD CE FB DC EA (9) The three ratios are traced out as we move aroundthe triangle in an anticlockwise direction. (Here we have AFBDCEA). It does not matter at which vertex we start, or in which direction we proceed; the product is always 1. Hence if we know two of these ratios, we can calculate the third one. Proof of Cevs Theorem We consider the two cases, namely, D is between B and C, and the other, D is not between Band C, together. (See Figures 1 and 2). .from (3) (DCA) (6DA)-(BD0) from (8) DcA)—(Dco) ~~ 80 _(80A) a DC (COA) (Note the above result holds for any point 0 on any Cevian AD from A. If O = D, we get equation (3).) 108), 5 HE BOO nacho (2) we nt numerators and denominators.) Hence larly 7 AF BD CE _ (AOC) (BOA) (COB) FB DC EA (BOC) (COA) (AOB) completing the proof of the theorem. Ceva's theorem is valid as long as AO, BO and CO, extended if necessary, meet the opposite sides. 12.6.3 Aninteresting Cevian identity For each point O in the interior of a triangle, we canassociate the sum of the “Cevian ratios”, namely, AO, 80. CO stich are referred to in Exercise (2) above. The Cevian ratios forthe centroid G of a OD” OE OF triangle, namely, the point where the three median: meet, are well known. Firstly they are all equal, and secondly, they are equal to 2. So the Cevian sum when O = Gis 6. It turns out that there is no other point in the interior of a triangle that has equal Cevian ratios, and moreover, all other Cevian sums are greater than 6. A The sum of the Cevian ratios associated with an arbitrary interior 7 point of a triange is at least 6. Equality is obtained at the centroid of the triangle, andonly at the centroid. Moreover, the centroid is the only point in theinterior of a triangle that divides the Cevians in the same ratio. o c Proofs vet = pee, y=. ay cova's theoren ajfy =1. Then the sum ofthe Cevian ratios, from po’ ba” FB Exercise (2), is AO, BO CO_AE,AF, BD , BF CED 0D” OE OF EC” FB’ DC” FA” EA LB 1 Lt feysrastapet B y a 8 {and this happens if and only if c= 8=/=1. Then BD = saying that AD, BE and CF are medians and O = G, the centroid, ‘Suppose now that the Cevians are equal. Let k be the common value. So AO _ BO _co OD OE OF = 243k The only non-negative solution is k= 2. And then Lipoid ped 1c, CE = EA and AF = FB which is the same as xk 30 as before, we conclude that O = G, the centroid B ye 12.7 APPLICATIONS OF LESSON 12 37 [si [aaa as) 207 [as [aa Yaa 6s ng LESSON 13: MANY PATHS, ONE DESTINATION In this lesson, we demonstrate how a single problem can be solved in a variety of ways, using tools from different areas of the FET curriculum. Consider the following: PROBLEM: Given the equations ay yexte (1) Determine the value or values of eso that (d) There are exactly two pairs of numbers (x; y that satisfy both the equations. (e) No pair of numbers satisfies both equationssimultaneously. (f)_ There is exactly one pair of numbers (x; y) that satisfies both the equations. (A) THE ALGEBRAIC APPROACH PRELIMINARIES: Let us first try to understand the question by substituting different values of c and see what happens. Let us start with c= 1. We have Pay a2 yextd To solve, we can “get rid” of y in the first equation, by replacing the y there with y=x+1 x +(e) =2 xx 42e41=2 2x? +2x-1=0 Now we have one equation in one unknown. This isa quadratic equation, for which the two values of x can be obtained by using the formula: abe vb? dae 4 ~b-vb?= 400 2a 20 Herea= 2,6=2ande= - 1. Substitu +2 4A) og 2(2) eVI2 2-12 4 4 =0,366 or 1.366 20 Substitute these values in y=x+c=x+1 to get (x;/)=(-0,366;0.634) & (~1,366;~-0,366) . Summarising, for c=1, there are exactly two solutions, so c= 1 isone of those values of c that answers the question (a) above. We need to find all such values of c. Let us try If we repeat what we did above (do it!), we come to 2x? +6x-+7=0, and, using the formula, we get Now, positive. So for c: (b) above. We need to find all such values of c. impossible to find a real number whose square is - 20, since the square of any real number is there are no solutions. $0 ¢= 3is one of those values of c that answers the question Let’s not specify values for c, and see what happens. Substitution of y=x+¢ into the other equation of (1) gives us: XO +bchd 2x +2xe+ (2? - n2e+ Vlecy - ate =2) a orn) Note that the number whose square root is calculated is 16—4c”, and its value for c =1 is 16-4.1" = 12, while for c=3, its value is 16 ~ 4(3*) = -20. Since 12 >, it had a square root and we concluded that there were two solutions, and since~ 20 < 0, which does not have a square root so we concluded that there were no solutions Hence: If 16~4c? >0 there will be exactly two solu:ions and if 16~4c? 0. (We know that c= 1 works). To solve this problem, we factorise the left hand side, that is, we write the left hand side as a product. 16-4¢’ =4(4-C) =4(2—-cJ2+c)>0 Now, a product of two numbers is positive if either both numbers are positive, or both numbers are negative. Let's see what happens in the first case. We have 2-c>Oand2+ c>0. Thatis 2>cande>-2, So cis a number greater than ~ 2 but less than 2. Draw the number line to see what's going on here mi NOTE: The numbers -2 and 2 are excluded in the darkened segment above. ‘Summarising: For all the numbers on the number line indicated above, that is, for ~2.2orc<-2, there are no solutions. m4 Using coordinate geometry Since OC is perpendicular to AB, (radius perpendicular to a1 _- i ——=—=-1, so equation of OCis tangent, Moc fe My 1 =X To find coordinates of C, solve y =-x and. simultareously.—x=x+6 x lies on tne circle, so 2or -2.etc ms LESSON 14: THE INFINITUDE OF PRIMES 14.1 IDENTIFYING PRIMES ‘Some years, like 2003, 2011 and 2017, are prime nunbers. One can use a calculator or a mathematical Package, to verify this. ‘An economical way of testing whether a given numter n is prime is not, is given by the following 14.1.1 Suppose pis not a factor of n for every prime number p less than or equal ton Then nis a prime number. For, suppose suppose n has the above property. We show that it must be prime. If not, it has a prime factor p Vn and “is Pp an integer. But 7 is an integer greater than 1, and w it has a prime factor, say q. It follows that qis a P 2° <= Jp, contradicting the given information. Our assumption is povn therefore false. That is, n isa prime number. 14.2 TO IDENTIFY ALL PRIMES LESS THAN A GIVEN NUMBER Now let us look ata related problem, namely, to find all the primes less than a given number. Let us say that we want to find all the prime numbers less thar 100. prime factor ofr. But q< To do this, we list all the numbers from 1 to 100. 1213 |4 15 |6]7 [8 |9 |10 ai [a2 | a3 | 14 | 15 | a6 | 17 | a8 | 19 | 20 21 | 22 | 23 | 24 | 25 | 26 | 27 | 28 | 29 | 30 31 | 32 | 33 | 34 | 35 | 36 | 37 | 38 | 39| a0 ai | 42 [43 | 44) 45 | 46 | 47 | 48 | 49 | 50 51 | 52 | 53/54) 55| 56 | 57|58| 59| 60 61 | 62 | 63 | 64 | 65 | 66 | 67 | 68 | 69 | 70 71 | 72 [73 75 | 76 | 77 | 78 | 79 | 80 Zi 81 | 82 | 83 | 84 85 | 86 | 87 | 88 89 | 50 91 | 92 | 93 | 94 95 | 96 | 97 | 98 | 99 | 100 26 is not a prime (remember, primes are by definitior all greater than 1), so delete it. Our strategy is to locate the primes, by deleting all tie multiples of the primes. Let us start with 2. Delete all the multiples of 2, but not 2 itself. a ]2]3 ]4 [5 Je |7 [a |9 |a0 31/42/13) a4/1s|as/17| 38/19] 20 21 | 22/23 | 2a] 25 | 26 | 27| 28 | 29| 20 31 | a2 | 33 | 24/35 | a6 | 37| a8 | 39 | 40 ai [42 [43/44] 45 | 46 | 47 | 48 | 49] 50 51 | §2/53| 84] 55/86/57] 58 | 59| 60 61 | 62 | 63 | 64 | 65 | 66 | 67| 68 | 69| 70 71 | #2 | 73| 74] 75 | 35 | 77| 78] 79| 80 1 | 82 [83/84/85 | a6 | 87 | aa | 89 | 90 91 [92 [93/94/95 | 96 | 97 | 98 | 99 | 400 We do the same for 3. Leave 3, but delete all the multiples of 3. [30 | a1 [32/13/34] 35 | a6 | 17[ a8] 19] 20 2a | 22 [23/24/25 | a6 | 27 | 2a) 29] 30 31 | 22/23/24 | 35 | a6 | 37| 38/29 | 40 aa | 42 [43 | 44) 45 | 46 | 47 [48 | 49 | 50 a [52 [53/54] 55 | 56 | 57 [52 | 59| 60 61 | 62 [63 | 64/65 | 66 | 67 [68 | 69 | 70 71 | 92 | 73 | 34 | 95 | 36 | 77| 38/79 | 80 aa | e2 [83/94/85 | e6 | a7 | aa | 89] 50 91 [92 [93/94 | 95| 96 | 97| 98 | 99 400 27 5 is next. This is easy ~ two columns are deleted. Then delete the multiples of 7. a ]2]3]4 {5 Je |7 Ja [9 [a0 ai | 42/13 | aa] as|ae|17| 48/19] 20 aa | 22 | 23/24) 25/26 | 27/28/29] 30 31 | 22 | 23 | 24 | 25 | a6 | 37| a8 | a9 | 40 ai [42 [43 | 44) 45 | 46 | 47 | 48 | 49] 50 ba [ 52/53/54) 55/86 |e [5a | 59] 60 ei | 62 | 63 | @4| 5 | 66 | 67 | 8 | 60] 70 71 | 32 | 73 | 74] 95 | 36 | 37 | 38/79] 80 84 | 82/83/84 85 | 86 | 87/88 | 89 | 50 ‘91 [92 [93 | 94/95 | 96 | 97 | 98 | 99 | 400 We are finished! For the square root of 100 is 10, and 2, 3, 5 and 7 are the only primes less than 10. The survivors in the above table are the primes numbers less than 100. Let’s erase all the crossed out numbers to see what we have got. The numbers that remain, namely: 2, 3,5, 7, 11, 13, 17, 19, 23, 29, 31, 37, 41, 43, 47, 53, 59, 61, 67, 71, 73, 79, 83, 89, 97 are the primes numbers less than 100. There are altogether 25 primes less than 100. You will notice that the last, namely the 10" line, has only one survivor. The twenty-first line has no survivors. (All the numbers from 201 to 210 are composite.) 14.3 THERE IS NO LARGEST PRIME NUMBER Long ago, the following question was posed: If we delete all the composite numbers as we have done above — that is, not just those which are less than some number, but all - surely, from some poirt onwards, all the rows will be blank rows? After all, at the start (that is with 2) all the even numbers areeliminated, that is, half disappear, then one third of them disappear, when we take the multiples of 3, and then one fifth, when we take the multiples of 5, and so on, then surely, after some time, maybe a ve'y long time, all the numbers beyond that are eliminated? If that is the case, the last survivor will be the largest prime number! So, is there a largest prime number? 28 ‘The answer is no. There is no largest prime numbet. ‘Another way of saying this is: ‘Think of any prime number. Then there is a prime number larger than that prime number. Yet another way of saying this is: 14.3.1 There are infinitely many prime numbers. All this is well and good. But is there any way of actvally proving this somewhat wild claim? Yes, there is. 14.3.2 Proof that there is no largest prime number ‘Take any prime number p. We will show that there ‘s a prime number larger than p. Consider a new number, namely, the product of all he numbers from 1 to p. Call this new number n. n=l. ». Now n+ 1 obviously has at least one prime factor. Call this prime factor q. Then g (which is a factor of n + 1), cannot also be a factor of n. (Consecutive numbers cannot have ‘common factors other than 1.) Now, every number less than or equal to p is a factor of n, since n is the product of all these numbers. So q cannot be less than or equal to p, for itis not a factor of n. ‘This means that g (which was chosen to be a prime number) is greater than p. ‘And this is what we set out to prove, namely, that there is a prime number greater than p! ‘Summarising: no matter which prime number you can think of, there is a larger prime number. This beautiful proof, which is exquisitely logical, is accredited to the great Euclid, a Greek mathematician who lived 2 300 years ago, and who also founded geometry as we know it n9 LESSON 15: USEFUL THEOREMS IN GEOMETRY 15.1 PARALLEL LINES The lines are parallel Alternate angles are equal: b=e,h=d Corresponding argles are equal: b=,h=g,a=d,c=e Co-interior angle: are supplementary: b +d=180", h+e=180". 15.2 ANGLES OF A TRIANGLE x+y+z=180" (sum of angles of a triangle) we=x-+y (exterior angle is equal to the sum of the interior opposite angles). © 15.3 ANGLES OF A POLYGON ‘The sum of the angles of a polygon ‘The sum of the exterior angles of 2 having n sides is n—2 straight angles, that polygon is 360 degrees. That is, is, x+y-424-~ = 180(n~2) degrees. a+b+c4d+~-=360 degrees. (See 6.1.3) 380 15.4 THE ANGLE WHERE BISECTORS MEET a-90 1320 ze-tea 15.5 CONGRUENCY A triangle has six properties (or numbers) associatec with it, the three sides and the three angles. Given two triangles, the triangles are said to be congruentif the six properties of the one are equal to the six properties of the other. The triangles are then identeal in every respect. (1) (5,5, 5): If three sides of one triangle are equal to three sides of another triangle, then the three angles of the one are equal to the three angles of the other. More precisely, the angles opposite these (equal) sides are also equal. is ” ZX (2) (s,2,5)): If two sides of one triangle, togetter with the angle included by these two sides, are equal to two sides and the included angle of another triangle, then the remaining three properties are also equal, that is, the triangles are congruent, B1 EX os XY A Inone case, and in one case only, the angleneed not be included. This is when the angle is a right angle. (3) (right angle, hypotenuse, side): If in two right-angled triangles, one side and the hypotenuse of ‘one triangle are equal to a side and hypoteruse of the other, then the remaining three properties of the triangle are equal. (4) If two angles and a side of one triangle are equal to two angles and the corresponding side of another triangle, then the remaining three groperties of the one triangle are equal to the remaining three properties of the other. oe NM XLS IX & oR Ln - Ss 15.6: ISOSCELES TRIANGLES 15.6.1 : i /\ -/\ /\ - J\ A right-angled isosceles triangle having side x, asin figure, has hypotenuse x2. Its angles are 45°, 45° and 90° 15.7 PERPENDICULAR BISECTORS 15.7.1 Any point on a perpendicular bisector is equidistant from the endpoints of that line segment. Thus in the figure PA = PB. 15.7.2 If AB = AC in triangle ABC, then A lies on the perpendicular bisector of BC. 133 /\ - Js 15.8 KITES Vv = WY 15.8 PARALLEL( RAMS: A parallelogram is a quadrilateral whose opposite sides are parallel. 15.8.1: A if ly if re equal, 15.8.2: A quadrilateral is a parallelogram if and onlyif its opposite sides are equal [FL 15.8.3: quadrilateral is a parallelogram if ans only ifits diagonals bisect each other a 1B4 15.8.4: A quadrilateral is a parallelogram if and only if two opposite sides are equal and E7-By 15.9 THE MIDPOINT THEOREM ining the midpoints of two si Is parallel to the thi . Moreover, its | isone half the length of the third side. A a 1 . E + DF ==AC ZN ~ ZN e 7 8 co 15.10 AREA FORMULAE 15.10.1 The area of a ractangle is the product of its base and height. A=bh. Le/L£ / 15.10.2 The area of a parallelogram is also the product ofits base and height. A= bh 385 15.10.3 The area of a triangle is half the base times the height: are: 15.10.4 The area of the triangle is jobsing. 15.11 THE THEOREM OF PYTHAGORAS Zexty’ Proof: Proof: Extend each of the sides holding the right angle, namely AC and AB, througt a length equal to the length of the other side, as shown. Complete the parallelogram ADFH by drawing parallel lines at Dand H. Since AD = AH =x +y and angle Ais a right angle, ADFH is a square. Now locate points E, P and B on the sides so that AC=DE=FP =x Then CD = =BA=y, as shown, Since A, D, F and H are all right 136 angles the four triangles in the figure are therefore congruent, all having the same area, namely In triangle ABC, a-+ 8 =90', so examining the angles 2:8, C, E and P we see that BCEP is a rectangle. But all the sides are equal (to 2), by congruence, soit actualy a square, having area z*, So the area of the large square is, on the one hand the sum of the five inside areas, and on the other the square of x + y. That is 2 +4(2xy)=(x+y)?. Simplify and subtract 2xy from both sides to obtain 2° =x" +y",, as required. Conversely, if we know that the square on the hypo:enuse is equal to the sum of the squares on the other two sides, then we can conclude that the triargle is right-angled. ea 4+y¥'then ZA 15.12 CIRCLES 15.12.1 The circumference ofa circle The most important property of a circle is that the distance from the centre to any point on the circle is always the same. This distance is called the radius o' the circle. The lenath of a circle is called the circumference of the circle. A line joining two points of a circle that noes through its centre is called a diameter. If dis the dianeter and ris the radius, then, since a diameter is composed of two radii, d = 2r. 37 ® For each of the circles above, we can divide the circumference C by the diameter d. That is, we can c calculate | Remarkably we will find that all threeanswers are the same! It does not matter how large or small the circle is. The ratio s is the same is all cases. In other words, the ratio of the circumference 2 of circle to its diameter isa constant . The constasts approximately —, which as a decimal number i approximately 3,1428. The actual value is closer to 2,1416, This number is called pi (the Greek name for the letter p), which is written as 7 . Thus we have: Replacing d with 2r, we get C=2ar that is, the circumference of a circle is equal to 2. times the radius. 15.12.2 Angle ina semicircle While on the subject of diameters, the following is of interest: If AB is a diameter of a circle and P is any point on the circle, then angle API is always a right angle. 138 1.12.3 The area ofa circle We all know that the area of a circle is A=zr*, where ris the radius of the circle. Now, we shall try to see where this formula came from. To begin with, we divide the circle into a very large number of triangles all having the centre of the circle ‘asa vertex. The area of the circle is approximately equal to thesum of the areas of all these (very) small triangles. ‘The area of each triangle is (; base times height). For a really “thin” triangle the “base” is almost equal to is the same for all the nw ingles, so the arez of the thin triangle in the figure is approximately 1 1 br So, when we are adding the areas ofall these tiangles, the quantity 51 comes out as a common factor, and the quantity in brackets in the sum of allthe bases of these small triangles, which, in the figure, is the sum of all the “b”s. But the sum of the 2's is again, approximately equal to the circumference of the circle, which, as we saw, is 27. Hence 1 Area of circle = > r(2zr) (Of course, this is just an indication, not @ complete sroof,) 15.13 TANGENTS TO A CIRCLE Note that a straight line can cut a circle at no place at all (see the figure), or at exactly two points (see CD in the figure). the figure), at exactly point (see AB in A tangent is a straight line that cuts a circle at exactly one point. So in the figure above, AB is a tangent to the circle. This point is called the point of contact. 89 ‘Tangents have some interesting properties. OO 15.13.1 The radius of a circle is perpendicularto a tangent to the circle at the point of contact. CY CY 15.13.2 outside the circle have equal lengths, 15.13.3 From a point A outside a circle having having centre O , draw the tangents AT and AB The quadrilateral ABOT is > MY Sesame (c) OA is the perpendicular bisector of BT. (see 15.6.2). 15.14 CIRCLE THEOREMS 15.141 Let A and B be two points on a circle having centre 0. Let P be any other point on the circle. Then ZAOB=2ZAPB mo QOS A) ©) es 15.14.2 Definition of“ subtends” Let AB be a line segment and P any point not on the line segment. We say that ‘AB subtends angle at P. 15.14.3 A diameter subtends a right angle at every point on the circle, 15.14.4 Angles in the same segment are equal Let A and B be two points on a circle. Let P and Q betwo other points on the circle on the same side of AB. Then ZAPB = ZAQB. m1 PP? gy -B This is true because all indicated angles are equal toone half of angle AOB, where 0 is the centre of the circle, 15.145 ‘There is always a circle containing the three verticesof a triangle. The same cannot be said for a quadrilateral. if it happens that the four vertices of ¢ quadrilateral lie on a circle, then the vertices are said to he cancyclic and the quadrilateral is said ta he a cyclic quadrilateral 15.14.6 The opposite angles of a cyclic quadrilateral are supplementary, => x+y=180° 15.14.7: ye v-Y m2 15.14.8: Ifa line segment subtends equal angles at two points on the same side of it, then the owe and therefore 15.12.10 applies. 7 15.14.9 ilaterd cyclic. os {\->- : > 15.14. cl a t angle inthe alternate segment, = M3 15.14.11: Equal chords subtend equal angles at the circumference, 15.15 APPLICATIONS OF LESSON 15, 21 2 23 34 35 7 39 az a5 52 [ar 34 100 105 a | 113, 114 115 17 164 167 175, 179 182 188 | 191, 200) 223 254 4 THE SOLUTIONS ® | SOLUTION | 3 -1=(° ays +1) = 242. 204=2.121.4.61 2 | Multiply all by 100000: 20130, 20100, 2010, 2013, 2000. The smallest is 0,02 Z| increase = 58—S0=8 8 Percentage increase = —-x100= 16% 2 | Four times a number of cattle minus twice the same number is twice that number. But this is 92. Therefore there are 46 cattle. v Draw a diagram and count: There are 6 x9 = 54 desks. OR: There are 344-1 rows and 4*6-1 columns and so there are 6x9=54 desks. 5 | Ifthe ratio of two numbers is given, say a :b, then the numbers themselves are ax and bx. So Pieter gets 7x and Jacob gets 5x sweets. The difference is 7x~5x= 14, so x=7 and the sum oR = 2 represents 14, so 1 represents 7 andthe total 7 + 5 = 12 represents 12x 7 = 84. 7 | €D+ EF =20 and BC + DE=25. The perimeter AB +BC + CD +DE +EF +A, B+ (BC + DE) + (CD + EF) 10+ 25 + 25 + 20= 90. | Write both fractions with denominator 5 3% 3 77g aNd gg” $0 the numerator of the fraction lies between 32 and 35. The number is ==. z By Pythagoras (Lesson 15.11), the 3 side is 8. The area can be 1 gh m4 calculated in ewoways: 5-68=510h, Thus h=“. Ms OR Let x and y be the lengths of the segments thet make up the hypotenuse. Then: KAY EW css (I) We =8'—-x' =6'—y' (Pythagoras) 2 ~y' =64-36=28 (x-y)ix+y)=28 from (1) 2) (2) + (2): 2x = 12,8 x=6,4 HP =(8—x)(8+x)=(1,6)(14,4)= 16144 4.12 24 10°10 3 oR cos6= © 10 1 | ifwe add the numbers 21, 23 and 26, we woud have counted all Mary's toys exactly twice, So she 21423424 ; - has “72274 35 toys, which means that ste has 35 21 =14 jets or Let}, tand c be the number of jets, teddy bea's and cars respectively. Then: te iz it Add both sides 2+ t+ c}-21 +234 26 s0j+t+e=35, j=Gttte)-(t+e)=35—21= 14. | The number is even (any multiple of 18s a multiple of2), 0 bis even, thatis, b=0, 2,4, Gor It is also a multiple of 9, o the sum of the digits is divisible by 9. (See 1.5). That is a+b+27, hence a+b, is divisible by 9. Since a and b are digits we have either a+b= 18 or a+b=9. In the first case, , which cannot be since b is even. Hence (0, ) = (9,0), (7, 2) or (5,4) oF (3,6) oF (1,8). Naximum difference is 8. A possible arrangement is GRGRGG We need only count the number of ways the red beads can be placed So the question is: in how many ways can two spaces be filled if there are six spaces available? This is the same as asking how many twoelement ubsets a six-element set has. (See Lesson 5.2) (6 There are ff '=15 different ways. oR Count: M6 RRGGGG, RGRGGG, RGGRGG, RGGGRG . RGGEGR. GRRGGG, GRGRGG, GRGGRG, GRGGGR GGRRGG, GGRGRG, GGRGGR GGGRRG, GGGRGR GGGGRR There are 5 with Rin first position. If Ris in sezond position, there are 4, and so on. In all, there are 5+4+342+1=15 possibilities. 15] Shoba takes 100 minutes to dust 100 ornaments, so on average, she dusts 1 ornament every minute. Shanti takes 200 minutes, 50 she dusts half an ornament in one minute. After 66 minutes, Shoba will have done 66 and Shanti 33. If Shoba does the last one, she will take one more minute, ‘making 67 minutes altogether. There were 365 days in 1985. So Tuesdays fellon the following days of the year: 1,8, 15, 22, 29, 36... Since 365 = 52x7 + 1, there are 52 full weeks in a year, and the last day (31 December) is the same as the first (1 January). Thus there are 52 + 1 = 53 Tuesdays in the year. Alternatively: The sequence 1, 8, 15, 22, 29, 3,...iS an arithmetic sequence with a= 1 and d= The number of Tuesdays n is the largest valueof n for which the n term is less than or equal to 365. + (n—1)d $365 1+(n-1)75365 134252 7 ns53 There are 53 Tuesdays. 3] 3001? = 302.3001 = 3001(3000++1) =9003000 + 3001 =9006001 has 4 zeros. oR Multiply out oR 30017 =(3000 +1)? =3000? +2.3000+4 = 9000000 + 6000 +1 = 9006001 ete. 15 | By the rule for divisibility by 11, (7 + 2) -dis dvisible by 11. So 9-d=0, 11, 22,.. Since dis a digit, d=9. 1] Rewrite as (2-1) + (4-3) +~ + (6-5) + +(50 - 49). How many one’s are there? Its the same as the number of numbers in 2, 4, 6, 50. Answer: 25 lake n= 1000 which isa er. Then 272 = 20% which tely equal to > ake n = 1000 which isa large number. Then“ =205, whichis approximately equal to > ro 2k There are 40 tiles of which (8+8+3+3) = 2 areat the edge. Probability is 7 | Let C=3x, Then A= 2x Since B and Care supplementary, we have B+ C= 180, so B + 3x = 180, or B = 180 - 3x. Sc D = 2B = 2(180 - 3x) =360 - Gx. But A*B#C+0 = 360 so 2x + (180 - 3x) + 3x + (360 - 6x) = 360, leading :0 x = 45 SoC = 3(45) = 135 degrees. M7 gee © Clearly PC=5. Since BC is parallel:o AD, ZPCA=ZCAD=@. But then APC is a triangle. 2 SOA The numbers in pos 2014 = 6.350 +4 The number at position 2010 is 1, at 2011 it is4, at 2012 it is 2, at 2013 itis 8, so at position 2014 it iss. ions 6, 12, 18, 24, ... are all 1 Let r be the radius of the small circle, and R the radius of the large circle. The therefore 2ar & 2a. The circumferences differ by 6, so 66.44 42 21 = approximately 5p BOT aap approximately. circumferences are 2n(R—1)=6 That is 2x(R-r)=6.Hence R- Remark: Suppose the radius of the Earth is r. Imagine a string wound tightly around the Earth. Its length would be 22. If the length of the string is is increased by 6 metres, and rhe lengthened string is formed into a circle so that its height above the ground is the same throughout the length of the string, how high is the string above the ground? If Ris the radius of the larger circle, the answer is R~r. We have just seen the this its value is almost one metre, a surprising result! 2345 9 2 1 3'4'56°"100 100 50 (all cancel each other, except 2 and 100). Weight of fuel when full is 58 — 34 = 24, When two-thirds full, the fuel weighs two-thirds of 24, that Is, 16. So tank then weighs 34+ 16 = SOkg. The sum of the 12 numbers is 12 x 18 = 216, while the sum of the smallest and largest is 2 x 28 = 56. So the sum of the other 10 numbers is 216 ~ £6 = 160. Their average is therefore 160/10 = 16, A played twice, and so was in the final. Since A was beaten by B, it follows that A and B were in the finals and A beat D in the first round. So B and C were teamed in the first round, and C lost, since B was in the final. So B beat C. Look rather at 6, 12, 18, Now 1000 = 6. 166 + 4, so there are 166 multisles of 6 that are less than 1000, the last being 6.166 = 996. So 5,11, 17,.., 995 are the numbers, and there ae 166 of them, Alternatively: The sequence is arithmetic, witi a = 5 and common difference d = 6. We need to determine the number of n such that the n™ term is less than 1000. a+(n—1)d <1000 546(n—1)<1000 n-15 99 21653 6 n<1662 Answer's 166. mg B let Aa=x & Zi =y. We try to calculate x-y. The two triangles are isosceles so we have a 2 But then, since the sum of the angles of a triangle is 180, we have, abetyty=180 xty=90" For f = 5, misat least 1, ie. m1 For f = 10, mis at least 1, i.e.m>2. In general, i. mat that is, Sm> f a Group from 4 to 9, 10 to 19, 20 to 29, and so on. The contributions of these groups to the number of digits are 9, 20, 20, 20, 20, 20, .. so the 100th digit isin the 6th group, that is, from 50 to 59. The last 1 is at 51. The number of I's altogether is therefore: 411414141 41-16. {All groups have one 1, except the second, which has one as a units digity and ten as tens digits.) The factors are of the form 11°13°17°19", where a, b, c and d can only be 0 or 1. (For example (a, b, ¢, d) can be (0,1,1,0) or (1,1,0,1)). There are 22.2.2 = 16 such choices, so 16 factors. Note that the increase is a fixed amount, not a fixed percentage. It does not matter how much there was in the beginning, so we can let the value start at 100, and the first increase is 10% of that, which is 10. At the end of the first, secord and third years, the value is therefore 110, 120 and 130. So the increase during the third year is fram 120 to 130. The percentage increase is. 10 100 20,100 _ 3 339, yo" 1 9% Let ZBPC=y be as indicated. AD = BC (since ABCD is a parallelogram), so triangles ADP and BCP are isosceles. Also, 2 ZAPD= 180 - 2x, so ZAPD=S0'-x.. ° “ @ Since BP 180", that is, 180° — 2x. Adding the angles at P, we get (90° —x)+72° + (180° - 2x) = 180°. So x=54° (C, ZC=y. But angles Cand D are o-interior angles. Hence 2x + |The number of arrangements of n objects is 12.3.4.....0 = n!. Answer is 1.2.3.4.5 = 120. mg We can use the given rule tofillin the blanks.At the last stage, 80=(2x'y)(Sxy'), xy" =8, xy=2. Z The portion of the paper that is folded is the right angled triangle CEF. Since CGF =90", we have by Pythagoras, CF* =CG" +GF*, thats, a 2=1+GF" so GF = 1, But then CGF is isosceles, so ZGFC =45' But EFC isaright angle. So ZEFD=180-90-45=45'. since ZD=90, ZDEF = 45 hencetriangle EDF is also isosceles. So d= DF, and V2 =DF +FG=d+1. Hence d= V2 -1. 38 | Method 4: Twice the length of the small rectangle = 3 times its breadth, 50 its length is 1,5 times its breadth. But the length and breadth add up to 15. The breadth is therefore 6 and the length is 9. Each small rectangle has area 54. The five together have area 5 x 54 = 270, which is the area of the new rectangle. Method 2: Using symbols, 3a = 2b, a + b=15,lead to Sab = 270. 2 Starting from the top left triangle, number them from 1 to 111, moving in a clockwise direction. Let the rectangle have . » dimensions 4a and 3b. We calculate the areas of trangles 1,3,5,7,9, and 11 | » 1&3: base a, height x y 5&7: base b, height 4a 9 & Li:base a, height y ‘Sum of areas = . * ” * % (ax + ax +4ab + 4ab tay + ay) =4ab + a(x + y). But x + y = 3b. Therefore sum of areas = Jab. ai Job 7 The fractions 2-2 | The areas of the four shapes are 6, 7, 4 and 5. The sum of the areas, being the area of a rectangle, must have 3 as a factor. Only 5 +4=9 and 5 +7 =12 qualify. 9 is a square, and clearly Fig. 3 cannot fit into Fig. 4. But Fig. 2 and Fig. 4 can make up a 3x 4 rectangle. oR By trial & error we can see 2 & 4 fit. 150 RC = RE? +EC* SORC=5. But the diagonals of a rectangle bisect each other. SoRK=% RC=2,5. 2013" — (2013 1)(2013+1)=20122014 The first three are less than ¥4. We need to compare only the last two. But 0,625 so the largest number is 0,625. Barbie’s bag can hold 60 balls. When it is twothirds full, it has 3(60)=40 balls. So (the excess) 10 is half of what Ken had in the beginning, which means he must have had 20 balls. ZDAC =103' (alternate angles) ZACD =180' —(103 +56")=21" {angles of a triangle) There are 12 possibilities: H1, H2, H3, H4, H5,H6, Ti, T2, T3, 74, TS, T6. So the probability of obtaining T6is > R Each side of the small square has length 56+28=2.cm, so each area is 4 cm’, Number of squares is 214345) +7 25.x4 =100 cm’, 5, so the area is ‘Suppose the area of each block in the top rowis 3. The three blocks in the second row together have area 12, so each has area 4, So area of black blocks is 2x342x442%3=20. Area of fl 12 = 36. Fi herefore == ea of flags 3x 12 = 36, Fraction is therefore 5==5 Remark : If the area of each block in the top row is taken to be 1, then the fraction is (of dts of 4}+(? ofa) _4+$ i248 5 2 “a6 3" What is on the floor at X? The same number that faced the right at C (which also faced right at B). But this number must have been opposite 3 a: the start. So 4 (that is, 7~3) is on the floor. And. therefore, when the die is at X, 3 is at the top. Alternate: Track the exposed faces. (1,2,3) > (24,2) > (2,5,4) > (2,1,4) (23,1) S03 is at the top. 151 The average of 125 and 140 is 132%, so, since125 is closest, the number is one of 125, 126,.., 132. The average of 142 and 121 is 131%, and, since 142 was closer than 121, the number is one of 132, 133, 134,... The answer is 132. ®t | =2.2.3.3.3.7m. For the number to be a perfect square, the exponents in the prime factorisation on the right must be even. So m must containan odd number of 3's and 7's in its factorization. Least m is 3x7 =21. 2 1 1 The area ofthe triangle is base times height = 1 while the area ofthe circle 2 tae is mr? Theratiois 57:11 =1:27. 2 | Method 1; The excess of Q over P, namely, 7Olitres, was achieved at the rate of 6 — 4 = 2 litres per minute, so the water must have been flowingfor 35 minutes. The volume of each container is 35x 4-+ 60 (or 35 x6- 10) = 200. Method 2: After t minutes, volume of water in Pis 4t and in Qis 6t. Let V be the volume of the each container in litres. Then Vt = 60 and 6t -V= 10, Add: 2t = 70, s0 t= 35.$0V= 60 + 4t = 60 + 4(25) = 200. |The shortest distance is the length of the straight line AB. Clearly ais the point (4; 0). ABs the hypotenuse of a right angled triangle. One side is the y-coordinate of B, which is the y- coordinate of (21; 12), hence 12, Suppose 8 has co-ordinates (b; 12). Then b +12 = 21, so b=9. The other side of the right-angled triangle is b- 4 = 9— Hence AB =5? +12? =169. AB = 13 xx =4x—3K4+x7 = 12 x +x-12=0 (x+4)(x-3)=0 Two solutions. Group the 1's into fours. The quotient has 6 ones, and there are 3 zeros between every pair of consecutive 1's. Answer: 5x3= 15. OR The quotient has 6 ones, the rest being zeros. The quotient has 24 ~ 3 digits. So there are 24~ 3-6 = 15 zeros. Substitute (2) in (3) 2d+d=18,s0d=6. From (3), a+b+c=12. Substitute (1) in (4) a+b+(2a+b)=12and 3a+2b= 12, 3a = 2(6~b) is positive and both a and b and are integers. So 6~ bisa multiple of 3. But 6 is a multiple of 3, hence b is a multiple of 3, that b = since 6-b > 0, we have b=1,2,3,4or5. 6.9, Also, 152 Hence b = 3. Substitute in (5): 3a + 6 = 12, so = 2. From (8), c= 7. Alternatively: Use equations (1)-(3) to solve for a and b in terms of c. 2a + B= Coremsnesnes(1) atb+c=12 Soa+b=12—C...(6) (a) ~ (6); 2(6)-(2):b Since a and b are positive integers, ¢> 6 andc<8. Hence ¢=7. Let u=x+2y, v=2x-+5y, Then u and vare irtegers and the inequality becomes u’ +(v. so (uv) e{(-1,0), (-2,2), (0,0), (0,2), (1,0), (1,1)} Solving for x and y in terms of u and v, we get x=Su-2v, y=v—2u, Substituting for u and, we obtain six solutions for (x, y, namely, (-5,2), (7,3), (0,0), (2,1), (5,-2) and (5,—1). Let x and y be the number of teachers and prefessors respectively. We need to find ~ . since 3! y the average age of the teachers, their total ag2 is 35x. Likewise, total age of professors is SOy. So the total combined age is 35x + SOy. But the average age of all taken together is 40. Hence. 35x +50y =40(x+y) 5x=10y x y ® | Notice that itis easy to multiply by 125, since 125 x8 = 1000 and 125 x 4 = 500. So let x= 125. Then 123? 129-124 x 125 x 126 = 24 A} fx 1)xlx +1) x8 12K416-x8+x = 16 - L1x= 16 11.125 = 16 - 1375 = -1359. "| We could have, for example, the situation below. The total number of games played is 144 23 +8 + 6+4+24141=59. Players | Standbys | Games 28 [32 14 Each game has exactly one loser, and every loser is knocked out 46 {0 23 and loses exactly one game. So there is 1-1 correspondence 16 (7 8 between “ganes” and “losers”. That is, the number of games is wz [3 6 exactly the seme as the number of players who lost. But there is 8 1 4 exactly one player who did not lose any game, namely, the winner 4 1 2 of the tournament. So there are 59 games. 2 1 1 2 0 1 @ ED gpg SEDs 1539 153 Clearly the sum of two integers is even if and anly if either both are even or both are odd. The product is odd if and only if both the numbersare odd. (If either of the numbers is even, the product is even) : In mod 2, even numbers are congruent to 0 and odd numbers are congruent to 1. So in iseven In mod 2, the equations become: (a) ma =1. (b) min =1.1.1=1 (o) m+n4i=tet+i-1 () 2m+3ne5 =0.1+1.1+1-0 (e) 2mén=0.141=1 So only (a) is even. Method 1: Ifthe integer nis even, then it hasthe form n = 2k, so m= 4k =.0.k?=,0, 0 when nis even, n? leaves a remainder of 0 when divided by 4 If mis odd, nis of the form n= 2k+1s0 r= d+ dk+1 =(040+1) a1 so the only possible values of r are 0 and 1. Check: The first 9 squares are 1, 4, 9, 16, 25, 36, 49, 64, 81 and the remainders on division by 4 are 1,0,1,0,1,0,1,0,1, 1. We write 2000 as a product ab of twonumbers having the same parity, and use the identity w=) -2) ‘ab(a O.and 1 +1 k(L+!)=54—1=> Lis an integer. So 1 + lis Method 2: (k+1)(/-+1)=k+/-+k+1=54+1=55. So k +1 is a factor of 55. Its one of 1, 5, 11 or 5S. Ifit is 1 or 55, we have that either k or lis 0, and we given that both k and are positive. So (k+4, 141) = (5, 11) o (11, 5) and k-+142 = 16. Hence k+1= 14. (a) The number is in the sequence 3, 13, 23, 33, 43,... as well as the sequence 4, 17, 30, 43, Answer is 43. (b) This problem cannot be solved by inspection, and needs a different approach. Let n be the required number. Then n=25a+11=36b+12. Subtracting, we get 36b-25a=1. We use lessons 2.2 and 2.7. 11=3(3)+2 3=2Q) +4 1=3~2(1)=3-(11-3(3)) = 4(3)-11=4(25-112))-11 =4(25)-9(11)= 4(25) -9(36 -1(25)) = 13(25)-9(36) Hence (a, b) = (13,9) and n=25a-+11=25(13)+11=336 Now let (x,y) be any solution of n = 25x+11 = 36y +12. Then 25x ~36y = 1 = 25(13) ~ 36(9), so 25(x— 13) = 36( y—9). 25 and 36 are coprime, and 36 is 2 factor of 25(x ~ 13), we can conclude that 36 is a factor of x ~ 13.That is X= 13 = 36k, so n= 25( 36k + 13) + 11= 900k + 336, where kis an integer. Clearly then, 336 is the smallest positive integral soluticn. +7 We need to determine the positive integers nfor which is an integer. ns3 +7 _nln+3)-3n+3)416 a3 ns3 =n-3+78, a3 oR (Long divide _n? +7by n + 3 to obtain quotien: n-3 and remainder 16. Then 157 1 +7=(n-3)(n-+3)+16 . Divide both sides by n-+3) The left hand is an integer if and only if ais an integer. That is, n-+3is a factor of 16, with n>0. 1n+3.€{4,8,16} giving three values for n, namely, 1, 5 and 13. $22 a ZF are all integers). (che (check: 7.5 Let (a, b,c) be a Pythagorean triple. Then a +6 = @. Let the remainders of a,b, and ¢, on division by 3, ber, s and t respectively. Then mod 3, a=r, b=s, andr+s? = 0 Since ris one of 0, 1 or 2, is one of O, 1, or4 so mod 3, ris O or 1. The same can be said of s and 2 is not possible, So one of rand + Hence, we cannot have rand s both equal to 4, since then t” sis zero, that is, either a or bis multiple of 3. As for 4, we do the same, but divide a, b and cby 4, to obtain remainders r, sand t respectively The remainders can be 0, 1, 2 or 3, and their squares, namely, 0, 1, 4 and 9 are congruent to 0 or 3, mod 4. Again, we cannot have r and s both equal to 1, so either ror s is a multiple of 4 Now divide a, b and ¢ by 5 to obtain remainders r, s and t respectively. The remainders can be 0, 1, 2,304, and their squares, namely, 0, 1,4, 9and 16 are equal to 0, 1 and 4 mod 5. (This means that every square number is congruent to one, 1 or 4 mod 5). If either r or s is zero we are done, for then ar b is divisible by 5. Otherwise, we have that a+b = =, (141,444 oF 1+4)=, (2,3 00) Neither 2 nor 3 is possible. So in this case, c isa multiple of 5. For example, if 1, 1,3,1,5, 3,7, 1,9, 5, 11, 3, 13, 7, 15, 1, thesum being 86. For the sum is 22 4, the highest odd factors of 1, 2, 3, 4,5, 6, 7,8, 9, 10, 11, 12, 13, 14, 15, 16 are it can easily be checked that In general, we have 1,2, 3,4, Syony 2" vo) Firstly, note that, by factoring out the highestpower of 2, every number n can be written as n=2"d, where dis the largest odd factor of r (Note that for any m in the sequence (1), n =2"d <2", sod<2'"~1, since d is not even.) We need to calculate d for each of the 2‘ numsers in the sequence (1), and add them. The method will be to collect, for each index m, all the associated d’s and add them. m=0,d.<2°: These are the numbers in the colection that have no even factors, that is, all the odd numbers, 1,3,5, Fu! 1 a) The number of terms in this sequence is the same as the number of terms in 2,4,6,8,...2 which is the same as the number in 1,2,3,4,..2", andthisis 2"! Alternatively we may use the formula for the n'*term of an arithmetic sequence. From Lesson 8.2.2, the sum of the first n odd numbers is n?. So the sum of the numbers in (1) is (ye? m=1,d<2™: These numbers are of the form “2 times an odd number.” They are: 158 2, 6, 10, 14,.., (2° —1). The sequence of thelargest odd factors for this collection is: 4,3, Byocuy PEL 2) hhas 2"? terms. Note that this is the same as (1), but with r replaced by r— 1. The sum of the numbers for m= is 2"*, Moving on to m=r—2, d< 2"\"=4, we see that the sum of the highest odd factors is 1+3 = 27. =1,d<2, so d=1 the only odd multiple of 2“ less than 2'is itself. The same applies to m Hence the sum of all the highest odd factors i 27 4294 4278 4-44-4141, each term corresponding to one of the r + 1 values of m. PUPA pot edt =24(4442 44 44a) The series in brackets is @ geometric series having r-1 terms and first term 4. The required sum is therefore ala 4) 442 3 3 2 +2 4g as Cheek: For r=3 and r=4, we have =86, as expected. Let x = PB = QC= RD =SA, Then 2x = BQ = CR = DS = AP. By Pythagoras, PQ? = BQ? + BP? =(2x)" +(x)’ =Sx" , but PQ’és the area of square PORS. The area of ABCD is, therefore (2x tx) =9x° So Area of PORS _Sx?_5 ‘Area of ABCD 9x7 9 Lexx be the number. Then the four numbers are xox x47 x48 So the sum is 4x-+ 16 which is equal to 312. 4x = 296 and x= 74, Let us use N, E, W, Sto denote the directions, and name the rooms as indicated. Then x a fle Neatb+e, E=c+dte +hegand S=gtfte, where each of N, E, Wand Sis atleast 3, since all rooms are occupied. From the given information, S=3W=2E=4N, NowN 23, But N 2 4 is impossible since then $216 and then the total number of people is at least 16 + 5 = 21, and we have only 20 people altogether. So N= 3, making 159 afi [i we4 x E=6 f s=12 But E+W=10and E+W+1+ = total number of people = 20. S01+f=10, and f=9. Since S = 12 and f=9, we have g+e=3 so git1or 2, Both values produce solutions. aji[a a[a 2/x[3 ax 1/9|2 2/9 [a The angle in a semicircle isa right angle. So Z°=90. Let a = BC, b = AC. We need to determine the aerimeter a+ b + 10, so we need to find a + b. By Pythagoras, a? +b®= 100. Also the area of the triangle is 11. That is, “ay orb Hence, (a + b)? =a" b? +2ab = 100+ 2(22) = 144 s0a+b=12and the perimeter a +b+10= 27 units. (a) 12 cannot be a factor of the odd number 2x - 3. oR If there is a solution x, multiply on both sidesby 6 to get 0 = 6(mod 12), contradiction. (b) 3 cannot divide 3x -7. Hence, neither can 12. oR multiply both sides by 4 to get a contradiction (c) Multiply by 2 to get a contradiction. (d) x=9 isa solution. (e) Multiply by 6 to get a contradiction. Alternatively: (a) 3 + 12k is always odd. No solution (b) 7 +12k cannot be a multiple of 3, no solution (c) Same. No solution (d) 5.52 1 mod12 so x = 45 mod 12 =9 med 12 gives infinitely many solutions ...9, 21, 33, .. (e) 10x~5 ends in 5 and therefore cannot be a multiple of 12. Only (d) has a solution. 160 ca ‘Method 1: When Sumayya covers 80 metres Fareeda would have run 100 metres. Since Sumayya actually had a 20 metre head start, they would meet at the point that is 80 metres from the actual start. We can suppose then that they are runring a race of 20 metres together, that is, one-fifth of 1 the original distance. She would there be =(20)=4 metres ahead ‘Method 2 Race 1, tend: aco 2, start FOS 100 End 4 Since Sumayya covers 80 when Fareeda covers 100, Sumayya’s speed is - th the speed of Fareeda’s. If, on the second run, Sumayya finéhes first, she would have run 100, while Fareeda 5, would have run = (100)=125 metres, But thatwould have made Fareeda fist, a contradiction. 0 4 Fareeda was first and Sumayya ran ~(120)=6 metres, 4 metres behind Fareeda at the winning post. The location F of the fire station is along BC, with BF = %. The distances are then %, 7% and 74, the maximum distance being 7%. The maximum ‘or any point along BA is greater than 8. For a point G on AB to better 7%, AG has to be less than %, aut then the maximum is greater than 11% . For any G on BC, if BG is less than or greater than ¥, the maximum is greater than 7. Alternatively: Suppose the point x in questior is along AC. For example, if AX = 4 and XC = 8, the maximum is the maximum of (4, 8, 4#7 = 11} which is 11. We can reduce 11 by moving X closer to ‘A. For example, if AX = 3, the maximum distance is the maximum of {3, 9, 10), which is 10. Evidently, the best point on AC would be that point X where XC = XA+7.S0X 1 willbe exactly half way on the bent line BAC. Then 7 + AX = >(7+12) 1 2 So the best point X on AC is such that AX = 2%, giving a maximum distance of 9%, The “minimum of the maximum distances” or each of the other two lines, are, similarly, 1 1 (g+12)=10 & =(7+8)= get ) 37 +8) - So the last is best; the point X on BC with BX = % ha greatest distance equal to the maximum of (%, 7%, 74}= 7%.lProblems like this are called minimax problems.] Let x be the age of the child. Then x-+3=a" and x—3=a where ais a positive integer. So a=(0+3) +3, @-0-6=0, 0= 30r-2. The child's age is x= a +3= 6. ‘Method 1: Note that the indices in the numerator are consecutive numbers, and the same holds for the denominator. The question is therefore: what must the bottom be multiplied by to get the top? Answer 2° = 8. 161 Method 2: 242424 2242427) 242+?) _ 56 ~Ps2e1 7 10° —1=99 has 2 digits. Likewise 10 —1has :01 digits, all of which are 9's. The sum is 101 x9= 809. 31] Each side of the small square has length 2, so zhe radius of each circle is 1. Aside of the large square has length 2 x diameter = 4. Its area is 16. S| From 25 to 99, there are 99-24 = 75 number: and 150 digits. From 100 to 208 there 208-99 = 109 ‘numbers and 327 digits. So there are 327 + 150 = 477 digits all together 3 | The number is a multiple of 5 and so ends in Gor 5. Therefore B is O or 5. The sum of the digits is divisible by 9, so A+ B + 28s divisible by 9. SoA + B is among 8, 17, 26,.. But A+ Bis ess than or equal to 9 +5 = 14. Hence A+B =8. 3 | Method 1: An examination of the expression reveals that its in the form a? ~2ab+b* =(a—b)*50 the answer is (2013-2000) =13° =168. Method 2: Let a = 2000. Then the expression & equal to (a+13¥ -2a(a+13)+0" =o" +26a+169-20" -260+0" =169 3 | There are as many terms in this expression asthere are numbers in 2012, 2008, 2004,.., 28, 24. Divide each by 4, we get 503, 502, 501... 8, 7.6. So there are 503 ~ (6-1) = 498 terms. Take them in pairs and sum to get 4+4+4+-+ = 249 x4 = 996, = 1 Each rangle i divided into 8 equal triangles. The largest shaded triangle has area (81)=9. The aya three smaller triangles have a combined area 2f (3))en =3. Answeris9+3=12. a The worst possible case occurs when the first colours removed are: 5 (yellow or white), 9 of each of the blue, geen and red. The next one will ensure that she has. ten of the same colour. The answer is 5+9+9+9+1=33, 162 2-3" =F -(3'? = 3)(2" +3") =55 The first factor is the smaller than the secondso we have 2-3) Sand From the first two, we obtain, by adding: 2'""=56 which is not possible since 56 is not a power of 2. From the second pair of equations, 2" =16, x+1=4, x=3, 3’ =11-8=3, soy=1 So (x;y) = (3; 1) is the only solution, Answe: one solution % | Regard the group having B, C and D as a single person, say F. Then A, F and E can be arranged in 1.2.3 6 ways. For each of these 6 ways, B, Cand D can arrange themselves in 6 ways. Answer is 6 x6 = 36 ways all together. 16 | We know PB = PQ and R= RC. The perimeter is AP+PRSAR = AP +PQ+QR + AR AP + PB RC AR AB+AC 2x AB 240. 101 For any number x, Foc+4)= flxlf(t)=2Flx) so (2013) =2(2012) =? f(2011)=2" (2010) =2" f(2013~m)=....=2™ f(2013-2012)=2™ F(a) = 102 If Bis the liar, then he speaks the truth when he says that one of them is lying. But he cannot tell the truth and be a liar at the same time. So B cannot be the liar. He speaks the truth so (a) Ais the iar (b) If xis their common age, 21 metres per minute Let A refer to the front of one train, and B thefront of the other. When travelling in opposite 164 directions, AB, the distance between A and Bis 0 at the start and x + at the end. A and 8 move away from each other at a speed of ~ + metres per minute, and in 9 minutes cover the distance 812 x+y. Hence, 2+ ) aD Substituting y = 150, we obtain x=, 74450 _soometres . R R 4y Solution 1 See 11.6 and 11.7 for an explanation of the above diagram, By the cosine rule , applied to triangle ABC, (x +3) =8? +(x +5) -2.8(x +5)cos60 24 6x49=644x7 +10x+25-Bx—40 Let r be the radius of the incircle. Its centre O s the point of concurrency of the angle bisectors, and. the radii are perpendicular to the sides of thetriangle at the points of contact. We need to calculate x. tan p= 2a+2B=1200, a+ f=60 +tane tana +tanB Ba 5+3V3tana eo ee and” a Stan” 3/5 Sane wi 9-5VStanae=5+3VStane tana=—— WB Hence ee 52.8 20, s00¢= 3. 165 188] Area of pentagon + sum of the areas of the foar triangles = Area of rectangle Using the % base times height formula, the sum of the areas of the triangles is (6x2 +3x2-44x4 45 x2) = 22 sq. cm Area of rectangle = 9x6 =54 ‘Area of pentagon = 54~ 22 = 32 sq.cm The trick isto paint the grid as if it were a chessboard, as in the diagram alongside. Note that the removed squares are both black, so we have 30 black and 32 white squares. But the 31 dominos will cover 31 black and 31 white squares! We conclude that itis impossible to place the dominos as required, TID | ft would take billion seconds. But how long i that? Let us do the ealeulation: 1.000 000 000 seconds = + minutes 60 ty hours 60.60 10” 4 60.60.24 “* 10° I _veaeg 60.60.28.365 32 years! We have to calculate h. Noter+h= R soh New circumference - Old circumference Dah 2ar 10 10, 70 2x 222 Which is about the height of an adult human, a very surprising result! RoI gene 166 12 | 10584000=1000.10584=10.10.10.4.2646 = 2.52.5.2.5.4.2.1323 25.25.25.2.2.2.9147=25.25,25,2.2.23.33.77 YIP The product contains two 7's. These can come only from 14. So there are two 14’s. The product of LISP the other numbers is, iaae 3°.5* =2'(15*)=16(15) So the number is 14.14.15.15.15.16 making six children altogether. (Another possibility is 12.14.14.15.15.20.) Alternatively: The product of five numbers fram the list is less than 20° =3 200000 <10 584000 and the product of seven numbers is greater than 1’ =121.121.121.11 > 11.100.100.100 > 10 #4 000. So the answer must be six. a c x 4 c Reflect A across the line and call the image, 8 Then AX + XC = BX + XC which, by the triangle inequality, it greater or equal to BC. 8 So BXC must be a straight line for the sum to be as small as possible. f ZAXD = 0, then since triangles AXD and BXD are congruent, 2BXD=0= ZCXE, thats, "the angle of incidence= the angle of reflection’. The point | therefore the point where 8C crosses the given line. 167 A gm 9 8 5 Reflect M across BC to M’. For any point P on BC, ; ain > ¢ ~ PRAM NE VT __ Let PA,PB, PCand PD be equal to x, a, b and c respectively. We nesd to determine x in terms of a, b and c. Draw lines © |q through P parallel to the sides of the rectangle. Name the segments p, 4, r ands as shown. We have (Observe that the hour hand moves through S minutes during the same time that the minute hand ‘moves through 60 minutes. So the minute travels 12 times faster than the hour hand. When the minute hand moves through x minvtes, the hour hand covers “~ minutes, so the number of minutes separating them is x— RD 12 '360°)=6' so the angle ax)g 14 between them (when the minute hand moves through x minutes) is (2) 7H dearees. w When they are at right angles to each other we have elther —~=90 or =~ 180 Far ay Minutes. The number of minutesbetween these two times ‘Method 1 (Geometrical Construct an equilateral triangle EGH, with H outside the square (see diagram below). 168 H Then ZCEH=15+60=75 Since CE = CG and HE=HG, EHGCisa kite. SoHC | perpendicular to EG. But BE is also perpendicular toEG. Hence HC||EB so ZECH = ZCEB=T7S , proving triangle HEC Is isosceles and HC= HE = EB. It follows that HEBC is a b rhombus. So BC= BE = BA. Clearly BC = CA, proving triangle | ABC is equilateral V\ Methoc 2 Let triangle ABD be equilateral with D in the interior of the square. ttis easy to show that 2DGE = ZDEG =15'. So the point C must be the same as the point D. Method3: If we reflect triangle BCE across EC, we get triangle BHC, and HEG = 75° ~ 15° = 60" so HEG Is equilateral. So HECG ‘A isakite,hence, since HC bisects opposite angles, HCE = 75°, so HEC, hence BCE is isosceles, etc. Use BE and BA to construct a reference frame, with B(0, 0), A(2, 0), E(0, 2) and Gl2, 2). Then x. =land tants =2 Ye =2-4 Now tangs’ —tan30__ 1-9 Tvtanas tan30 1+ 3-v3 _8-y3/’ =2-J3 343 9-3 —tan15' = V3 , from which we deduce tants =tan(45 -30' After simplification, we see tan15 and Ye ZABC =60" , etc. ‘Method 5 (Wali Rahman! Equation of CG; y-2=tan15'(x-2) so ye~2=tan45 (x, ~2) ‘Method 6 (Shan Pillay) We can assume the sides are all two units in length. Since AECB: 1 So CE cos 15 = 1, Hence CE=——* — . By the cosine formula, cosi5 169 |, 1+4cos15¢0875, cos? 15 |. Ledcosi5sin15 cos 15 1-2sin30 4 cos? 15 Hence BC= AC= AB , proving ABC is equilareral. We prove the assertion by contradiction, thatis, we assume itis not true, and eventually arrive at something known to be false. And then we can conclude that our assumption is false, proving the truth of the assertion, So let us assume that no three of the eighteer sums are equal. Note first that the only possible sums are 0, 1, 2, 3, 4, 5, 6, 7 and 8. None of these 9 numbers occurs as a sum more than twice. Indeed, as there are 18 numbers, each sum occurs exactly twice. The sum 0 occurs only when all eight entries are O. If itis a diagonal sum, then no row or column can have sum 8, a contradiction, Thus 0s attained as either a row ora columnsum. Without loss of generality, we can assume there isa row consisting entirely of zeros. (Column: can become rows after a 90 degree rotation about the centre of the grid.) ololololololojo Now 8 can be neither a diagonal sum nor a coumnn sum. So both the 8's are row sums. Therefore there are two rows that are made up entirely of ones. And then the second 0 is also a row sum. The sum 1 occurs twice. Wherever it occurs, there will be one 1 and seven 0's. All the columns and doToToloTo]olo}) thetwo diagenals have at least two 1's. So once again, it must be TTT Tea] we rows tha: have sum 1 Sate oT eT ora TS] ledk now at sum 7. Wherever this sum occurs, there is exactly Sretetoteteters| ened. $0 as tefore, both sums 7 must occur in the rows. With all 010101010} 01110) cignt rows accounted for, the remaining sums must occur in the columns or dagonals. ELE} 21212122) Now consicer the four rows whose sums are, 17,7 21141111212 TOL shaded inthe dagram, Two of these rows have exactly one 1 and the othe! two have exactly one 0. Consider the columns in Which these four numbers do not occur. There are at least four of them and all have sum 4! This contradicts the fact that sum 4 occurs exactly twice. 170

You might also like